ORTHOPEDIC MCQS WITH ANSWER SPORT 04

ORTHOPEDIC MCQS WITH ANSWER SPORT 04

1.       A 21-year-old collegiate wrestler sustains a blow to his right eye during a match.  Examination reveals anisocoria with a dilated right pupil.  The globe is properly formed, and extra-occular movements and the visual field are grossly intact.  What is the most likely diagnosis?

 

1-         Traumatic mydriasis

2-         Detached retina

3-         Dislocated lens

4-         Corneal abrasion

5-         Traumatic hyphema

 

PREFERRED RESPONSE: 1

 

DISCUSSION: Traumatic mydriasis occurs from a contusion to the iris sphincter.  This is a transient phenomenon during which the iris fails to constrict properly, resulting in a dilated pupil.  More severe trauma can result in a tear of the sphincter and permanent pupillary deformity.  In association with head injury, traumatic anisocoria would be a concerning indicator of the severity of injury.  Retinal detachment, lens dislocation, corneal abrasion, and traumatic hyphema are all potential results of eye injury but are not reflected by this clinical description.

 

REFERENCES: Brucker AJ, Kozart DM, Nichols CW, Irving MR: Diagnosis and management of injuries to the eye and orbit, in Torg JS (ed): Athletic Injuries to the Head, Neck, and Face, ed 2.  St Louis, MO, Mosby Year Book, 1991, pp 650-670. 

Orlando RG, Doty JH: Ocular sports trauma: A private practice study.  J Am Optom Assoc 1996;67:77-80.

 

 

 

2.       After making a tackle, a football player is found prone and unconscious without spontaneous respirations.  Initial management should consist of

 

1-         log roll to a supine position, helmet removal, and initiation of assisted breathing.

2-         log roll to a supine position, head and neck stabilization, face mask removal, and CPR.

3-         log roll onto a spine board, head and neck stabilization, face mask removal, and CPR.

4-         head and neck stabilization, log roll to a supine position, helmet removal, and initiation of assisted breathing.

5-         head and neck stabilization, log roll to a supine position, face mask removal, and initiation of assisted breathing.

 

PREFERRED RESPONSE: 5

 

DISCUSSION: The on-field evaluation and management of a seriously injured athlete requires that health care teams have a game plan in place and proper equipment that is readily available.  The initial step, which consists of stabilizing the head and neck by manually holding them in a neutral position, is then followed by assessment of breathing, pulses, and level of consciousness.  If the athlete is breathing, management should consist of mouth guard removal and airway maintenance.  If the athlete is not breathing, the face mask should be removed, with the chin strap left in place.  The airway must be established, followed by initiation of assisted breathing.  CPR is instituted only when breathing and circulation are compromised.  In the unconscious athlete or if a cervical spine injury is suspected, the helmet must not be removed until the athlete has been transported to an appropriate facility and the cervical spine has been completely evaluated.

 

REFERENCES: McSwain NE, Garnelli, RL: Helmet removal from injured patients.  Bull of Am Coll Surg 1997;82:42-44.

Vegso JJ: Field evaluation and management of head and neck injuries.  Post Grad Adv Sport Med 1987;10:2-10.

 

 

 

 

3.       A 23-year-old baseball pitcher who has diffuse pain along the posterior deltoid reports pain during late acceleration and follow-through.  Examination of his arc of motion from external rotation to internal rotation at 90 degrees of shoulder abduction reveals a significant deficit in internal rotation when compared to the nonthrowing shoulder.  Initial management should consist of

 

1-         a cortisone injection to the subscapular bursa.

2-         posterior capsular stretching.

3-         strengthening of the external rotators and scapular stabilizers.

4-         continued pitching and working through the pain.

5-         a sling and rest.

 

PREFERRED RESPONSE: 2

 

DISCUSSION: Loss of internal rotation is common among overhead throwers and tennis players.  Posterior capsular stretching can improve symptoms when accompanied by rest and gradual resumption of throwing.  To avoid a false impression of improvement, cortisone injection is not recommended.  Pitching through pain can cause further damage to the labrum and capsule.  A sling and external rotator strengthening will not improve internal rotation.

 

REFERENCES: Kibler WB: Biomechanical analysis of the shoulder during tennis activities.  Clin Sports Med 1995;14:79-85.

Jobe FW, Tibone JE, Jobe CM, Kvitne RS: The shoulder in sports, in Rockwood CA, Matsen FA (eds): The Shoulder.  Philadelphia, PA, WB Saunders, 1990, pp 961-990.

 

 

 

4.       A 54-year man has left shoulder pain and weakness after falling while skiing 4 months ago.  Examination reveals full range of motion passively, but he has a positive abdominal compression test and weakness with the lift-off test.  External rotation strength with the arm at the side and strength with the arm abducted and internally rotated are normal.  MRI scans are shown in Figures 1a and 1b.  Treatment should consist of

 

1-         arthroscopy and labral repair.

2-         arthroscopy and supraspinatus repair.

3-         arthroscopy and subscapularis repair.

4-         arthroscopy and supraspinatus and infraspinatus repair.

5-         open repair of the pectoralis major.

 

PREFERRED RESPONSE: 3

 

DISCUSSION: The examination findings are consistent with subscapularis muscle weakness but normal supraspinatus and infraspinatus strength.  The lift-off test and abdominal compression test are specific for subscapularis function.  The MRI scan reveals a chronic avulsion and retraction of the subscapularis.  The transverse image reveals a normal infraspinatus muscle, and the sagittal image reveals an atrophic subscapularis.  Surgical repair of the isolated subscapularis tendon is indicated.

 

REFERENCES: Iannotti JP, Williams GR: Disorders of the Shoulder: Diagnosis and Management, ed 1.  Philadelphia, PA, Lippincott Williams & Wilkins, 1999, pp 31-56.

Gerber C, Hersche O, Farron A: Isolated rupture of the subscapularis tendon: Results of operative treatment.  J Bone Joint Surg Am 1996;78:1015-1023.

 

 

 

 

5.       A 17-year-old high school long distance runner is seeking advice before running a marathon for the first time.  What advice should be given regarding his fluid, carbohydrate, and electrolyte intake around the time of the race?

 

1-         Restrict fluid intake 2 hours before the start of the race to avoid abdominal cramping.

2-         Drink low osmolality (less than 10%) solutions before, during, and after the race.

3-         Drink fruit juice, such as orange juice, instead of water to replenish essential carbohydrates.

4-         Drink high osmolality (greater than 10%) solutions before and during the race and low osmolality solutions after the race.

5-         Avoid the use of glucose polymers because they slow down gastric emptying and may lead to abdominal cramping.

 

PREFERRED RESPONSE: 2

 

DISCUSSION: The goal of fluid replenishment should be to replace the sweat that has been lost.  Sweat is mostly water, with a small concentration of salts and other electrolytes.  Absorption is enhanced by solutions of low osmolality.  Scientific research has also shown that adding carbohydrates to the drink improves athletic performance.  Carbohydrates such as glucose and maltodextrins (glucose polymers) stimulate fluid absorption by the intestines.  Fructose slows intestinal absorption of fluids.  Drinks that are high in fructose, such as orange juice, can lead to gastrointestinal distress and osmotic diarrhea.

 

REFERENCES: Kirkendall D: Fluids and electrolytes, in The U.S. Soccer Sports Medicine Book.  Baltimore, MD, Williams and Wilkins, 1996.

Gisolfi CV, Duchman SM: Guidelines for optimal replacement beverages for different athletic events.  Med Sci Sports Exerc 1992;24:679-687.

 

 

 

 

6.       Figure 2 shows the radiograph of a 26-year-old auto mechanic who injured his right dominant elbow in a fall during a motocross race.  Examination reveals pain and catching that limits his range of motion to 45 degrees of supination and 20 degrees of pronation.  The interosseous space and distal radioulnar joint are stable.  Management should
consist of

 

1-         splinting for 3 weeks, followed by range-of-motion exercises.

2-         aspiration of the hemarthrosis, followed by range-of-motion exercises the following day.

3-         fragment excision.

4-         open reduction and internal fixation.

5-         radial head excision.

 

PREFERRED RESPONSE: 4

 

DISCUSSION: The radial head is an important secondary stabilizer of the elbow, helping to resist valgus forces.  There has been a movement toward open reduction and internal fixation of the radial head when technically feasible, especially in a relatively high-demand athlete or laborer.  The examination and radiograph suggest that displacement of the fragment is great enough to create a mechanical block.  Extended splinting would only serve to encourage arthrofibrosis.  Early range of motion is appropriate if there is minimal displacement of the radial head fragement, it is stable, and there is no mechanical block to motion.  Fragments larger than one third of the joint surface should be excised only if it is not possible to reduce and repair the fragment.  Primary excision of the radial head should be avoided if possible.  Complications after excision of the radial head include muscle weakness, wrist pain, valgus elbow instability, heterotopic ossification, and arthritis.

 

REFERENCES: Hotchkiss RN: Displaced fractures of the radial head: Internal fixation or excision?  J Am Acad Orthop Surg 1997;5:1-10.

Esser RD, Davis S, Taavao T: Fractures of the radial head treated by internal fixation: Late results in 26 cases.  J Orthop Trauma 1995;9:318-323.

 

 

7.       Figure 3 shows the clinical photograph of a wrestler who has an acute mass in his ear.  He does not wear protective headgear.  The area is mildly tender and without erythema.  Management should consist of

 

1-         observation.

2-         antibiotic therapy.

3-         irrigation and debridement.

4-         aspiration and compression.

5-         excision.

 

PREFERRED RESPONSE: 4

 

DISCUSSION: The patient has an auricular hematoma.  This injury is typically related to blunt trauma, occuring in wrestlers who do not use protective headgear.  The goal of treatment is to remove the fluid, reapproximate the perichondrium to the underlying articular cartilage, and limit reaccumulation of the fluid in attempt to prevent cartilage necrosis.  Aspiration and application of a compressive dressing offers the best chance to achieve this goal.  There are no signs of infection such as marked tenderness, erythema, or surrounding edema to justify antibiotic use or irrigation and debridement.  The mass does not warrant excision. 

 

REFERENCES: Kaufman BR, Heckler FR: Sports-related facial injuries.  Clin Sports Med 1997;16:543-562.

Griffin CS: Wrestler’s ear: Pathophysiology and treatment.  Ann Plastic Surg 1992;28:131-139.

 

 

 

 

8.       A patient with no history of patellar instability sustains a traumatic lateral patellar dislocation. What structure most likely has been torn?

 

1-         Vastus medialis obliquus

2-         Medial patellofemoral ligament

3-         Medial patellotibial ligament

4-         Medial retinaculum

5-         Quadriceps tendon

 

PREFERRED RESPONSE: 2

 

DISCUSSION: Any of the above structures may be involved in a lateral patellar dislocation. However, biomechanic studies have found that the medial patellofemoral ligament is the major soft-tissue static restraint of lateral patellar displacement, providing at least 50% of this function.

 

REFERENCES: Desio SM, Burks RT, Bachus KN: Soft tissue restraints to lateral patellar translation in the human knee.  Am J Sports Med 1998;26:59-65.

Conlan T, Garth WP Jr, Lemons JE: Evaluation of the medial soft-tissue restraints of the extensor mechanism of the knee.  J Bone Joint Surg Am 1993;75:682-693.

Warren LF, Marshall JL: The supporting structures and layers on the medial compartment of the knee: An anatomical analysis.  J Bone Joint Surg Am 1979;61:56-62.

 

 

9.       A 22-year-old swimmer underwent thermal capsulorrhaphy treatment for recurrent anterior subluxation.  Following 3 weeks in a sling, an accelerated rehabilitation program allowed him to return to swimming in 3 1/2 months.  While practicing the butterfly stroke, he sustained an anterior dislocation.  He now continues to have symptoms of anterior instability and has elected to have further surgery.  Surgical findings may
include a

 

1-         biceps subluxation.

2-         glenoid rim fracture.

3-         subscapularis detachment.

4-         loose body.

5-         deficient anterior capsule.

 

PREFERRED RESPONSE: 5

 

DISCUSSION: Complications of thermal capsule shrinkage or accelerated rehabilitation include capsule ablation.  Since the original surgery did not include labral reattachment, findings of a Bankart lesion or a glenoid fracture from a nontraumatic injury are unlikely.  Subscapularis detachment or biceps subluxation is a postoperative complication of open repairs.  Failure of early postoperative instability treatment should not produce loose bodies.

 

REFERENCES: Abrams JS: Thermal capsulorrhaphy for instability of the shoulder: Concerns and applications of the heat probe.  Instr Course Lect 2001;50:29-36.

Hecht P, Hayashi K, Lu Y, et al: Monopolar radiofrequency energy effects on joint capsular tissue: Potential treatment for joint instability. An in vivo mechanical, morphological, and biochemical study using an ovine model.  Am J Sports Med 1999;27:761-771. 

 

 

 

 

10.     A 12-year-old boy reports knee discomfort after prolonged strenuous activities.  He denies knee swelling or catching and has no pain with activities of daily living.  A radiograph is shown in Figure 4.  Prognosis for the pathology shown is most
influenced by

 

1-         weight.

2-         gender.

3-         the knee compartment involved.

4-         open or closed growth plates.

5-         limb alignment.

 

PREFERRED RESPONSE: 4

 

DISCUSSION: While many factors play a role in the outcome of osteochondritis dissecans, ample evidence has shown that the prognosis is most influenced by the growth status of the plates.  If the growth plates are open, the chance of a successful outcome is significantly greater than if they are closed.

 

REFERENCES: Federico DJ, Lynch JK, Jokl P: Osteochondritis dissecans of the knee: A historical review of etiology and treatment.  Arthroscopy 1990;6:190-197.

Linden B: Osteochondritis dissecans of the femoral condyles: A long-term follow-up study.  J Bone Joint Surg Am 1977;59:769-776.

 

 

 

 

11.     A 70-year-old golfer has pain in her dominant shoulder.  She reports that initially the pain was at night but now she is unable to play.  Examination reveals weakness in external rotation and shoulder abduction.  Radiographs reveal the humeral head articulating with a thin acromion.  Management should consist of

 

1-         a scapular and rotator cuff strengthening program.

2-         arthroscopy.

3-         review of her golf technique.

4-         humeral head replacement.

5-         an erythrocyte sedimentation rate.

 

PREFERRED RESPONSE: 1

 

DISCUSSION: Chronic rotator cuff tears should be nonsurgically managed initially with a strengthening program.  A cortisone injection may reduce inflammation.  Surgery is reserved for patients who continue to have pain and lose sleep despite the use of physical therapy.  Blood tests for infection or inflammation are nonspecific.  Arthroscopy may play a role, but surgical replacement is reserved for advanced cases.

 

REFERENCES: Bokor DJ, Hawkins RJ, Huckell GH, et al: Results of nonoperative management of full-thickness tears of the rotator cuff.  Clin Orthop 1993;294:103-110.

Wirth MA, Basamania C, Rockwood CA Jr: Nonoperative management of full-thickness tears of the rotator cuff.  Orthop Clin North Am 1997;28:59-67.

 

 

 

 

12.     Which of the following structures is the most important restraint to posterior subluxation of the glenohumeral joint when positioned in 90 degrees of flexion and internal rotation?

 

1-         Subscapularis muscle and tendon

2-         Supraspinatus muscle and tendon

3-         Middle glenohumeral ligament

4-         Inferior glenohumeral ligament

5-         Coracohumeral ligament

 

PREFERRED RESPONSE: 4

 

DISCUSSION: The posterior band of the inferior glenohumeral ligament is the most important restraint to posterior subluxation of the glenohumeral ligament with the shoulder in 90 degrees of flexion and internal rotation.  With the shoulder in external rotation, the subscapularis is an important stabilizer to posterior subluxation.  When the shoulder is in neutral rotation, the coracohumeral ligament is the primary stabilizer.  The middle glenohumeral ligament functions primarily to resist anterior translation of the shoulder in the midrange of abduction.  The supraspinatus muscle and tendon have relatively little contribution to anterior and posterior translation of the glenohumeral joint.

 

REFERENCES: Blasier RB, Soslowsky LJ, Malicky DM, Palmer ML: Posterior glenohumeral subluxation: Active and passive stabilization in a biomechanical model.  J Bone Joint Surg Am 1997;79:433-440.

Flatow EL, Warner JI: Instability of the shoulder: Complex problems and failed repairs: Part I. Relevant biomechanics, multidirectional instability, and severe glenoid loss.  Instr Course Lect 1998;47:97-112.

 

 

 

 

13.     A 50-year-old competitive tennis player sustained a shoulder dislocation after falling on his outstretched arm 3 weeks ago.  He now reports that he has regained motion but continues to have painful elevation and weakness in external rotation.  A subacromial cortisone injection provided 3 weeks of relief, but the pain has returned.  Which of the following studies will best aid in diagnosis?

 

1-         CT

2-         Limited bone scan

3-         MRI

4-         Joint aspiration

5-         Functional capacity examination

 

PREFERRED RESPONSE: 3

 

DISCUSSION: Based on these findings, the most likely diagnosis is a rotator cuff injury and probable tear; therefore, MRI is the study of choice.  CT is preferred for articular fractures.  A bone scan is nonspecific and can identify inflammation or occult fracture.  Joint aspiration is not likely to identify an effusion.  Physical therapy and a functional capacity examination are used to identify weakness during recovery prior to a return to work or sports.

 

REFERENCES: Hawkins RJ, Bell RH, Hawkins RH, Koppert GJ: Anterior dislocation of the shoulder in the older patient.  Clin Orthop 1986;206:192-195.

Matsen FA III, Thomas SC, Rockwood CA: Anterior glenohumeral instability, in Rockwood CA, Matsen FA III (eds): The Shoulder.  Philadelphia, PA, WB Saunders, 1990, pp 526-622.

 

 

 

14.     In the arthroscopic photograph shown in Figure 5, the structure labeled “A” functions primarily as a restraint to translation of the humeral head in what direction?

 

1-         Inferiorly with the arm adducted to the side

2-         Anteriorly with the arm abducted to 45 degrees and at neutral rotation

3-         Anteriorly with the arm abducted to 45 degrees and maximally externally rotated

4-         Anteriorly with the arm abducted to 90 degrees and at neutral rotation

5-         Anteriorly with the arm abducted to 90 degrees and maximally externally rotated

 

PREFERRED RESPONSE: 1

 

DISCUSSION: The superior glenohumeral ligament identified as “A” in the figure functions primarily as a restraint to inferior glenohumeral translation of the adducted arm.  The middle glenohumeral ligament is highly variable and pooly defined in up to 40% of the population and functions to restrain anterior translation of the externally rotated arm in the midrange of abduction.  The anterior band of the inferior glenohumeral ligament is the primary restraint to anterior/inferior translation of the head with the shoulder abducted to 90 degrees and in maximum external rotation.

 

REFERENCES: Ticker JB, Bigliani LU, Soslowskiy LJ, et al: Inferior glenohumeral ligament: Geometric and strain-rate dependent properties.  J Shoulder Elbow Surg 1996;5:269-279.

Owen MD, Kregel KC, Wall PT, Gisolfi CV: Effects of ingesting carbohydrate beverages during exercise in the heat.  Med Sci Sports Exerc 1986;18:568-575.

 

 

 

15.     During the anterior approach for repair of a distal biceps tendon rupture, what structure, shown under the scissors in Figure 6, is at risk for injury?

 

1-         Brachial artery

2-         Median nerve

3-         Posterior interosseous nerve

4-         Lateral antebrachial cutaneous nerve

5-         Antecubital vein

 

PREFERRED RESPONSE: 4

 

DISCUSSION: The most commonly injured neurovascular structure during an anterior approach for the repair of a distal biceps tendon rupture is the lateral antebrachial cutaneous nerve.  This structure is located lateral to the biceps tendon and in a superficial location just deep to the subcutaneous layer.  The antecubital vein is medial and superficial with the brachial artery and median nerve also medial to the biceps tendon but deep to the common flexors.  The posterior interosseous nerve is deep within the supinator muscle and can be injured in the deep dissection or through the posterior approach when using a two-incision approach.

 

REFERENCES: Kelly EW, Morrey BF, O’Driscoll SW: Complications of repair of the distal biceps tendon with the modified two-incision technique.  J Bone Joint Surg Am 2000;82:1575-1581.

Ramsey ML: Distal biceps tendon injuries: Diagnosis and management.  J Am Acad Orthop Surg 1999;7:199-207.

 

 

 

16.     Which of the following variables has been shown to have the greatest influence on the higher rate of anterior cruciate ligament (ACL) tears in women when compared to men for similar sports?

 

1-         Hormones

2-         ACL strength

3-         Notch width

4-         Neuromuscular training

5-         Leg alignment

 

PREFERRED RESPONSE: 4

 

DISCUSSION: All of the variables have been proposed as possible causes for the increased incidence of ACL tears in women versus men.  The general differences in the level of neuromuscular training however, specifically conditioning and muscle strength, have been shown to play the greatest role.

 

REFERENCES: Harmon KJ, Ireland ML: Gender differences in noncontact anterior cruciate ligament injuries. Clin Sports Med 2000;19:287-302.

Arendt EA: Knee injury patterns among men and women in collegiate basketball and soccer.  Am J Sports Med 1995;23:694-701.

Rozzi SL, Lephart SM, Gear WS, Fu FH: Knee joint laxity and neuromuscular characteristics of male and female soccer and basketball players.  Am J  Sports Med 1999;27:312-319.

 

 

 

17.     Figure 7 shows the MRI scan of a 23-year-old competitive rugby player who has anterior ankle pain and swelling.  He states that he has been playing for many years and has sprained his ankle several times.  Examination will reveal what specific hallmark feature?

 

1-         A palpable effusion

2-         Positive anterior drawer test

3-         Positive external rotation test

4-         Pain with forced dorsiflexion

5-         Loss of subtalar motion

 

PREFERRED RESPONSE: 4

 

DISCUSSION: The history and MRI findings indicate the presence of anterior tibiotalar osteophytes.  This is frequently observed in soccer, rugby, and football athletes who play on grass or turf surfaces and repetitively push off and change directions.  Examination may reveal an effusion but no loss of subtalar motion.  A positive external rotation (Klieger) test is described as pain at the distal ankle with external rotation of the foot and is observed in patients with syndesmosis sprains.  This patient may have an increased anterior drawer because of a history of sprains; however, this finding is not specific for anterior impingement of tibiotalar osteophytes.  The most specific finding on physical examination is pain with forced dorsiflexion.

 

REFERENCES: Ogilvie-Harris DJ, Mahomed N, Demaziere A: Anterior impingement of the ankle of the ankle treated by arthroscopic removal of bony spurs.  J Bone Joint Surg Br 1993;75:437-440.

Cannon LB, Hackney RG: Anterior tibiotalar impingement associated with chronic ankle instability. J Foot Ankle Surg 2000;39:383-386.

 

 

 

18.     A 21-year-old collegiate female cross-country athlete reports right hip pain that begins about 12 miles into a run, followed by pain resolution when she discontinues running.  However, each time she tries to resume a running program, she experiences recurrence of pain deep in the anterior groin.  A plain radiograph and MRI scan are shown in Figures 8a and 8b.  Management should consist of

 

1-         cessation of running and implementation of cross training until healing is complete.

2-         a bone scan, skeletal survey, and chest radiographs to assess for other sites of involvement.

3-         a radioisotope injection and scintinigraphic-guided percutaneous biopsy.

4-         percutaneous fixation with multiple cannulated screws.

5-         excision, bone grafting, and internal fixation.

 

PREFERRED RESPONSE: 1

 

DISCUSSION: The history is consistent with a stress fracture.  Findings on the plain radiograph are marginal, but the MRI scan shows evidence of stress reaction in the medial neck of the femur (compression side).  A lesion on the compression side is not normally at risk for displacement and usually can be managed nonsurgically.  A bone scan would further identify the lesion but is not necessary.  A skeletal survey and chest radiograph are used in staging a tumor.  Radioisotope injection and guided biopsy are sometimes used for osteoid osteomas.

 

REFERENCES: Boden BP, Osbahr DC: High-risk stress fractures: Evaluation and treatment.  J Am Acad Orthop Surg 2000;8:344-353.

Lynch SA, Renstrom PA: Groin injuries in sport: Treatment strategies.  Sports Med 1999;28:137-144.

 

 

 

19.     A 21-year-old football player who sustained a direct blow to the posterior hindfoot while making a cut is unable to bear weight on the injured foot.  Examination reveals tenderness and swelling of the great toe metatarsophalangeal (MTP) joint.  Radiographs are shown in Figures 9a and 9b.  What is the most likely diagnosis?

 

1-         Dislocation of the great toe MTP joint

3-         Rupture of the volar plate

2-         Fracture of the lateral sesamoid

4-         Fracture of the lateral sesamoid and rupture of the plantar plate

5-         Subluxation of the sesamoids

 

PREFERRED RESPONSE: 4

 

DISCUSSION: Turf toe occurs in collision and contact sports in which the athlete pushes off to accelerate or change direction and there is hyperextension of the great toe MTP joint.  Typically, there is also axial loading of the posterior hindfoot, which increases the hyperextension of the MTP joint.  The most common presentation is pain and swelling of the MTP joint and inability to hyperextend the joint without significant symptoms.  With significant force, fractures of the sesmoids and plantar soft tissues can occur.  The radiographs do not show a dislocation of the great toe MTP joint because it is concentrically located on both radiographs.  However, the radiographs show a fracture of the lateral sesamoid or a diastasis of a bipartite lateral sesamoid.  The medial sesamoid is also proximal indicating a rupture of the plantar (volar) plate.  Therefore, the most likely diagnosis is a fracture of the lateral sesamoid with rupture of the plantar plate leading to proximal migration of the proximal fragment of the lateral sesamoid and the medial sesamoid.

 

REFERENCES: Rodeo SA, et al: Diastasis of bipartite sesamoids of the first metatarsophalangeal joint.  Foot Ankle 1993;l4:425-434.

Rodeo SA, O’Brien S, Warren RF, et al: Turf toe: An analysis of metatarsal joint sprains in professional football players.  Am J Sports Med 1990;18:280-285.

 

 

 

20.    Examination of an 18-year-old professional soccer player who was forcefully kicked across the shin while attempting a slide tackle reveals a marked effusion and limited motion of the knee.  The tibia translates 12 mm posterior to the femoral condyles when the knee is held in 90 degrees of flexion.  There is no posteromedial or posterolateral instability.   Management should consist of

 

1-         early reconstruction of all injured structures.

2-         knee immobilization in 30 degrees of flexion for 2 to 4 weeks.

3-         knee immobilization in full extension for 2 to 4 weeks.

4-         protected weight bearing and intense hamstring strengthening.

5-         no weight bearing, followed by a gradual return to sports.

 

PREFERRED RESPONSE: 3

 

DISCUSSION: The patient has an acute grade III posterior cruciate ligament injury.  The majority of grade I and II injuries can be treated with protected weight bearing and quadriceps rehabilitation, and most patients can return to sports within 2 to 4 weeks.  In contrast, grade III injuries require immobilization in full extension for 2 to 4 weeks to protect the posterior cruciate ligament and the other posterolateral structures presumed to be damaged.  The mainstay of postinjury rehabilitation for all posterior cruciate ligament injuries is quadriceps strengthening exercises, which have been shown to counteract posterior tibial subluxation.

 

REFERENCES: Miller MD, Bergfeld JA, Fowler PJ, Harner CD, Noyes FR: The posterior cruciate ligament injured knee: Principles of evaluation and treatment.  Instr Course Lect 1999;48:199-207.

Posterior Cruciate Ligament Injuries in Principles and Practice of Orthopaedic Sports Medicine.  Philadelphia, PA, Lippincott, Williams and Wilkins, 2000.

 

 

 

 

21.     What type of injury is considered the major mechanism of cervical fracture, dislocation, and quadriplegia in contact sports and diving?

 

1-         Flexion

2-         Extension

3-         Flexion-compression

4-         Flexion-distraction

5-         Flexion-rotation

 

PREFERRED RESPONSE: 3

 

DISCUSSION: A compression or burst injury occurs with vertical loading of the spine, such as from a blow to the vertex with the neck flexed (eg, spear tackling in football).  This leads to vertebral end plate fractures before disk injury.  At higher forces, the entire vertebra and disk may explode into the spinal canal.  Analysis has shown this to be the major mechanism of cervical fracture, dislocation, and quadriplegia.  With the normal head-up posture, the cervical spine has a gentle lordotic curve, and forces transmitted to the head are largely dissipated in the cervical muscles.  When the neck is flexed, the cervical spine becomes straight, with the vertebral bodies lined up under one another.  This allows for minimal dissipation of the impact forces to be absorbed by the neck muscles. 

 

REFERENCES: Cantu RC: Head and spine injuries in youth sports.  Clin Sports Med 1995;14:517-532.

Proctor MR, Cantu RC: Head and neck injuries in young athletes.  Clin Sports Med 2000;19:693-715.

Torg JS: Epidemiology, pathomechanics, and prevention of athletic injuries to the cervical spine.  Med Sci Sports Exerc 1985;17:295-303.

 

 

 

 

22.    A 17-year-old high school football player injures his right ankle during a game.  Examination reveals swelling and a closed ankle deformity, with normal foot circulation and sensation.  Radiographs are shown in Figures 10a and 10b.  In addition to closed reduction, management should include

 

1-         cast immobilization.

2-         delayed fixation of the medial malleolus.

3-         immediate fixation of the medial malleolus and plating of the fibula.

4-         immediate fixation of the medial malleolus, plating of the fibula, and placement of a syndesmotic screw.

5-         immediate fixation of the medial malleolus and placement of a syndesmotic screw.

 

PREFERRED RESPONSE: 4

 

DISCUSSION: The examination and radiographs reveal a closed fracture-dislocation of the ankle with tibiofibular diastasis.  Immediate fixation of the medial malleolus and plating of the fibula are indicated.  If residual tibiofibular diastasis occurs with lateral translation of the fibula after plating, a syndesmotic screw is placed to stabilize the syndesmosis.  Ankle fracture-dislocations associated with a proximal fibular fracture (Maisonneuve fracture) require syndesmotic fixation, but the fibula is not plated.  Unstable ankle fractures require surgical treatment.  If swelling is severe (fracture blisters, loss of skin wrinkling), a compressive splint is applied and surgery is delayed for 5 to 7 days.

 

REFERENCES: Browner BD, Jupiter JB, Levine AM, Trafton PG: Skeletal Trauma.  Philadelphia, PA, WB Saunders, 1992, pp 1887-1957.

Stiehl JB: Ankle fractures with diastasis.  Instr Course Lect 1990;39:95-103. 

 

 

 

 

23.    Figure 11 shows a consecutive sequence of MRI scans obtained in a 12-year-old boy who has had increasing lateral knee pain and catching for the past 6 months.  Examination reveals pain localized to the lateral joint line.  Range-of-motion testing reveals a 5-degree lack of full extension on the involved side.  Plain radiographs and laboratory values are within normal limits.  What is the most appropriate management?

 

1-         Activity modification

2-         Hinged knee brace

3-         Partial meniscal excision

4-         Lateral release

5-         Physical therapy

 

PREFERRED RESPONSE: 3

 

DISCUSSION: Discoid menisci are rare causes of lateral knee pain in children.  Various etiologies have been proposed, including failure of central absorption of the developing meniscus and hereditary transmission.  Patients with discoid menisci have pain, clicking, and locking with a loss of active extension on range-of-motion testing.  Classification of discoid menisci according to the Watanabe classification include complete, incomplete, and Wrisberg ligament type.  The Wrisberg variant contains an abnormal posterior meniscal attachment.  MRI is the diagnostic tool of choice, revealing a thick, flat meniscus generally seen in three consecutive MRI images.  Symptomatic knees are often associated with a meniscal tear or degeneration and are managed with arthroscopic partial excision to a more normal shape (saucerization). 

 

REFERENCES: Vandermeer RD, Cunningham FK: Arthroscopic treatment of the discoid lateral meniscus: Results of long-term follow-up.  Arthroscopy 1989;5:101-109.

Bellier G, Dupont JY, Larrain M, et al: Lateral discoid menisci in children.  Arthroscopy 1989;5:52-56.

 

 

 

 

24.    A collegiate football player who sustained a blow to the head during the first quarter of a game is confused for several minutes after the hit but does not lose consciousness.  He had two similar episodes in games earlier in the season.  When should he be allowed to return to play?

 

1-         Immediately

2-         In the second half

3-         In 1 week

4-         In 4 weeks

5-         Next season

 

PREFERRED RESPONSE: 5

 

DISCUSSION: Using the traditional concussion grading scale, the patient sustained a grade I concussion because he did not lose consciousness and his abnormal cognitive level lasted less than 1 hour.  If this was the player’s first concussion, theoretically he could return to play later in the game provided that he had no confusion, headache, or associated symptoms.  However, because it was the third concussion for the year, participation in contact sports should be terminated for the season.

 

REFERENCES: Guskiewwicz KM, Barth JT: Head injuries, in Schenk RC Jr (ed): Athletic Training and Sports Medicine.  Rosemont, IL, American Academy of Orthopedic Surgeons, 1999, pp 143-167.

Kelly JP, Rosenberg JH: Diagnosis and management of concussion in sports.  Neurology 1997;48:575-580.

 

 

 

 

25.     For the athlete performing heavy exercise, the magnitude of core temperature and heart rate increase is most proportional to 

 

1-         water debt at the onset of exercise.

2-         sodium debt at the onset of exercise.

3-         potassium debt at the onset of exercise.

4-         ambient temperature.

5-         percentage of lean body mass. 

 

PREFERRED RESPONSE: 1

 

DISCUSSION: Studies examining the impact of graded water debt have clearly shown that the magnitude of core temperature and heart rate increase accompanying work are proportional to the magnitude of water debt at the onset of exercise.  Though added thermal burden from hot climates is a factor, it appears to be less significant.

 

REFERENCES: Latzka WA, Montain SJ: Water and electrolyte requirements for exercise.  Clin Sports Med 1999;18:513-524.

Montain SJ, Sawka MN, Latzka WA, et al: Thermal and cardiovascular strain from hypohydration: Influence of exercise intensity.  Int J Sports Med 1998;19:87-91.

Sawka MN, Young AJ, Francesconi RP, et al: Thermoregulatory and blood responses during exercise at graded hypohydration levels.  J Appl Physiol 1985;59:1394-1401.

 

 

 

 

26.    Initial repair of the large U-shaped rotator cuff tear shown in Figure 12 consists of closing the tear side-to-side to take advantage of margin convergence.  The most significant biomechanical consequence of this repair step results in

 

1-         increased strength of the rotator cuff repair by creating thicker repair construct.

2-         decreased size of the defect exposing the humeral head.

3-         decreased stress in the rotator cuff at the site of the side-to-side repair.

4-         decreased stress in the rotator cuff at the free margin and greater tuberosity interface.

5-         decreased stress in the rotator cuff crescent cable. 

 

PREFERRED RESPONSE: 4

 

DISCUSSION: Margin convergence refers to the phenomenon that occurs with side-to-side closure of large U- or L-shaped rotator cuff tears in which the free margin of the tear converges toward the greater tuberosity as the side-to-side tear progresses.  The creation of the converged cuff margin creates decreased strain in the free margin of the repaired cuff, resulting in a decreased strain in the repair sutures.  While the size of the humeral head defect is made smaller with side-to-side closure, biomechanically, this is less significant.  The mild increase in thickness of the repair at the side-to-side margin is less important than a reduction in stress in the repaired tissue.  Stress in the crescent cable region of the cuff actually increases and becomes more physiologic in transmitting force from the cuff to the greater tuberosity.

 

REFERENCES: Burkhart SS: A stepwise approach to arthroscopic rotator cuff repair based on biomechanical principles.  Arthroscopy 2000;16:82-90.

Burkhart SS, Athanasiou KA, Wirth MA: Margin convergence: A method of reducing strain in massive rotator cuff tears.  Arthroscopy 1996;12:335-338.

 

 

 

 

27.     A 15-year-old athlete collapses suddenly during practice and dies.  What is the most likely cause of death?

 

1-         Hypertrophic cardiomyopathy

2-         Atrial fibrillation

3-         Pulmonary embolism

4-         Ruptured aorta

5-         Mitral valve prolapse

 

PREFERRED RESPONSE: 1

 

DISCUSSION: The number one cause of sudden death in the young athlete is myocardial pathology, with hypertrophic cardiomyopathy being most common.  Because of cardiac muscle hypertrophy, the ventricular capacity is diminished and can result in decreased cardiac output.  During exertional activities, the increased demand may not be able to be met and leads to sudden death.  While the other choices can be the cause of sudden death in an otherwise healthy young athlete, their incidence is even more rare.

 

REFERENCES: Van Camp SP, Bloor CM, Mueller FO, et al: Nontraumatic sports death in high school and college athletes.  Med Sci Sports Exerc 1995;27:641-647.

Maron BJ, Shirani J, Pollac LC, et al: Sudden death in young competitive athletes: Clinical, demographic, and pathological profiles.  JAMA 1996;276:199-204.

Mills JD, Moore GE, Thompson PD: The athlete’s heart.  Clin Sports Med 1997;16:725-737.

 

 

 

 

28.     A 17-year-old football player continues to have discomfort after sustaining a blow to his midthigh during a game 8 weeks ago.  A plain radiograph is shown in Figure 13.  What is the most appropriate management?

 

1-         Immobilization

2-         Rest with range-of-motion exercises

3-         Steroid injection

4-         Excision

5-         Irradiation

 

PREFERRED RESPONSE: 2

 

DISCUSSION: The patient has myositis ossificans.  Rest of the involved area is important to help limit the continued irritation of the muscle, but range-of-motion exercises are important to limit stiffness.  While immobilization for 1 or 2 days following a muscle contusion is appropriate, longer periods of immobilization result in muscle atrophy and fibrosis.  Injections and irradiation have not been found to be of benefit for myositis ossificans.  Excision is rarely required, and if performed, it should not be performed prior to maturation of the lesion, which is a minimum of 6 months.

 

REFERENCES: Lipscomb AB, Thomas ED, Johnston RK: Treatment of myositis ossificans traumatica in athletes.  Am J Sports Med 1976;4:111-120.

Beiner JM, Jokl P: Muscle contusion injuries: Current treatment options.  J Am Acad Orthop Surg 2001;9:227-237.

Ryan JB, Wheeler JH, Hopkins WJ, et al: Quadriceps contusions: West Point update.  Am J Sports Med 1991;19:299-304.

 

 

 

 

29.    When standing, dorsiflexion of the great toe will accentuate

 

1-         midfoot pronation.

2-         heel valgus.

3-         internal tibial rotation.

4-         rigidity of the transverse tarsal articulation.

5-         parallel alignment of the talonavicular and calcaneocuboid joints.

 

PREFERRED RESPONSE: 4

 

DISCUSSION: Dorsiflexion of the great toe will accentuate rigidity of the transverse tarsal articulation.  Through the windlass mechanism, dorsiflexion of the great toe tightens the plantar fascia, stabilizing the longitudinal arch and placing the foot in supination.  Supination makes the talonavicular and calcaneocuboid joints nonparallel, accentuating the rigidity of the transverse tarsal articulation.  The heel also tends to go into varus, resulting in obligatory external tibial rotation.

 

REFERENCES: Mann RA: Biomechanics of the foot and ankle, in Mann RA, Coughlin MJ (eds): Surgery of the Foot and Ankle, ed 6.  St Louis, MO, Mosby, 1993, pp 1-44.

Hicks JH: The mechanics of the foot: II. The plantar aponeurosis and the arch.  J Anat 1954;88:25.

 

 

 

 

30.    A 26-year-old professional rodeo bull rider sustained a grade III midshaft femoral fracture after being thrown from his bull.  He underwent closed interlocking intermedullary nailing with a titanium rod, and his recovery was uneventful.  Prior to returning to competition, the patient must 

 

1-         be able to run and walk without pain.

2-         refrain from vigorous activity for 6 months.

3-         achieve full hip and knee range of motion.

4-         achieve symmetric lower extremity strength.

5-         have radiographic evidence of a circumferential external bridging callus.

 

PREFERRED RESPONSE: 5

 

DISCUSSION: While it is recommended that a patient gain full range of motion, pain-free function, and symmetric strength prior to returning to vigorous activities, it is absolutely essential that radiographs of the fracture site reveal a circumferential external bridging callus to prevent refracture.  This is particularly important for comminuted femoral fractures with various sized fragments.  It is also recommended that a return to rodeo riding be postponed for at
least 1 year.

 

REFERENCES: Brumback RJ, Ellison TS: Intermedullary nailing of femoral stress fractures.  J Bone Joint Surg Am 1992;74:106-112.

Bucholz RW, Jones A: Fractures of the shaft of the femur.  J Bone Joint Surg Am
1991;73:1561-1566.

Butler MS, Brumback RJ: Interlocking nailing for ipsilateral fractures of the femur, femoral shaft, and distal part of the femur.  J Bone Joint Surg Am 1991;73:1492-1502.

 

 

 

 

31.     A 19-year-old soccer player feels a pop in his knee while making a cut and notes the development of an effusion over several hours.  Examination reveals medial joint line tenderness, but the knee is stable to manual stress testing of all ligaments.  Examination under anesthesia confirms a stable knee.  What is the most critical factor in determining healing after repair of the lesion shown in Figure 14?

 

1-         Rim width

2-         Rim length

3-         Time from injury to repair

4-         Use of a fibrin clot

5-         Whether the tear occurred in the medial versus lateral meniscus

 

PREFERRED RESPONSE: 1

 

DISCUSSION: Numerous clinical and basic science investigations have evaluated meniscal tear characteristics to identify factors that either promote or mitigate against meniscal healing.  Complex tears have been noted to heal poorly, while longitudinal tears heal more predictably.  Tear length, time from injury to repair, medial versus lateral meniscal tears, and the use of a fibrin clot have not been shown to consistently affect meniscal healing.  However, rim width, the distance of the tear site from the peripheral meniscocapsular junction (vascular supply), has been shown to have a significant role in the ability of a meniscus repair to heal.

 

REFERENCES: DeHaven KE, Arnoczky SP: Meniscus repair: Basic science, indications for repair, and open repair.  Instr Course Lect 1994;43:65-76.

Henning CE, Lynch MA, Clark JR: Vascularity for healing of meniscus repairs.  Arthroscopy 1987;3:13-18.

 

 

 

 

32.    Which of the following tissues has the highest maximum load to failure?

 

1-         Native anterior cruciate ligament (ACL)

2-         Bone-patellar tendon-bone with a width of 10 mm

3-         Central quadriceps tendon with a width of 15 mm

4-         Quadruple semitendinosus and gracilis tendons

5-         Tibialis tendon allograft

 

PREFERRED RESPONSE: 4

 

DISCUSSION: All of the tissues noted above are stronger than native ACL.  Although it is often thought that the bone-patellar tendon-bone graft is the strongest when selecting a graft source for ACL reconstruction, biomechanical studies show that the quadruple semitendinosus and gracilis tendons are the strongest of the tissues listed.

 

REFERENCES: Woo SL, Hollis JM, Adams DJ, et al: Tensile properties of the human femur-anterior cruciate ligament-tibia complex: The effects of specimen age and orientation.  Am J Sports Med 1991;19:217-225.

Staubli HU, Schatzmann L, Brunner P, et al: Quadriceps tendon and patellar ligament cryosectional anatomy and structural properties in young adults.  Knee Surg Sports Traumatol Arthrosc 1996;4:100-110.

Wilson TW, Zafuta MP, Zobitz M: A biomechanical analysis of matched bone-patellar tendon-bone and doubled looped semitendinosus and gracilis tendon grafts.  Am J Sports Med 1999;27:202-207.

 

 

 

 

33.    A 20-year-old basketball player has tenderness and bruising after sustaining a blow to the knee.  A radiograph is shown in Figure 15.  What is the most likely diagnosis?

 

1-         Patellar fracture

2-         Patellar dislocation

3-         Bipartite patella

4-         Vastus lateralis tear

5-         Tumor

 

PREFERRED RESPONSE: 3

 

DISCUSSION: The patient has a bipartite patella.  The line between the fragment and the main patella is smooth and sclerotic, indicating a chronic, not acute, entity.  The location is classic for a bipartite patella, not a tumor.

 

REFERENCES: Schmidt DR, Henry JH: Stress injuries of the adolescent extensor mechanism.  Clin Sports Med 1989;8:343-355.

Weaver JK: Bipartite patellae as a cause of disability in the athlete.  Am J Sports Med 1977;5:137-143.

 

 

 

 

34.    Reconstruction of the posterior cruciate ligament (PCL) via the inlay technique involves exposure of the PCL tibial insertion site by a posterior

 

1-         lateral approach through the lateral gastrocnemius/biceps femoris interval.

2-         approach using the medial sural cutaneous nerve to split the medial and lateral gastrocnemius interval.

3-         medial approach between the medial gastrocnemius and semitendinosus muscles.

4-         medial approach between the medial gastrocnemius and semimembranosus interval.

5-         medial approach between the semimembranosus and semitendinosus interval.

 

PREFERRED RESPONSE: 4

 

DISCUSSION: The posterior medial approach through the semimembranosus/medial gastrocnemius interval is used in the inlay technique for PCL reconstruction.  Exposure of the posterior capsule of the knee through this interval provides the greatest margin of safety to avoid injury to the tibial nerve, motor branch of the medial gastrocnemius, and the peroneal nerve.  The direct posterior approach using the medial sural cutaneous nerve allows exposure of the popliteal neurovascular structures, but deep dissection through this interval places the motor branch of the medial gastrocnemius at risk.  The interval between the semitendinosus and semimembranosus is used in accessory incisions with medial meniscus repairs but does not allow exposure of the PCL insertion.

 

REFERENCES: Berg EE: Posterior cruciate tibial inlay reconstruction.  Arthroscopy 1995;11:69-76.

Hoppenfeld S, deBoer P: Surgical Exposures in Orthopaedics: The Anatomic Approach, ed 1.  Philadelphia, PA, JB Lippincott, 1984, pp 427-435.

 

 

 

 

35.     A 36-year-old recreational tennis player sustains the injury shown in Figure 16.  Management should consist of

 

1-         observation.

2-         rehabilitation.

3-         immobilization.

4-         primary repair.

5-         reconstruction.

 

PREFERRED RESPONSE: 4

 

DISCUSSION: The MRI scan shows a rupture of the patellar tendon.  This injury is most appropriately addressed with primary repair.  For athletic individuals, the results of nonsurgical management are suboptimal.  Reconstructive procedures are not necessary.

 

REFERENCES: Matava MJ: Patellar tendon ruptures.  J Am Acad Orthop Surg 1996;4:287-296.

Marder RA, Timmerman LA: Primary repair of patellar tendon rupture without augmentation.  Am J Sports Med 1999;27:304-307.

 

 

 

 

36.    Figure 17 shows the clinical photograph of a 45-year-old female tennis player who has right arm pain and weakness with elevation after undergoing a cervical biopsy several months ago.  The cause of her shoulder weakness is damage to the

 

1-         spinal accessory nerve, causing shoulder elevation with the scapula translated and the inferior angle rotated medially.

2-         spinal accessory nerve, causing shoulder depression with the scapula translated laterally and the inferior angle rotated laterally.

3-         long thoracic nerve, causing shoulder elevation with the scapula translated medially and the inferior angle rotated medially.

4-         long thoracic nerve, causing shoulder depression with the scapula translated laterally and the inferior angle rotated laterally.

5-         thoracodorsal nerve, causing shoulder depression with the scapula translated laterally and the inferior angle rotated laterally.

 

PREFERRED RESPONSE: 2

 

DISCUSSION: The patient has primary scapulotrapezius winging caused by surgical damage to the spinal accessory nerve during a lymph node biopsy.  Other causes include blunt trauma, traction, and penetrating injuries.  With spinal accessory palsy, the shoulder appears depressed and laterally translated because of unopposed serratus anterior muscle function.  With primary serratus anterior winging that is the result of long thoracic nerve palsy, the scapula assumes a position of elevation and medial translation with the inferior angle rotated medially.  The thoracodorsal nerve innervates the latissimus dorsi and is not associated with scapular winging.

 

REFERENCES: Kuhn JE, Plancher KD, Hawkins RJ: Scapular winging.  J Am Acad Orthop Surg 1995;3:319-325.

Wright TA: Accessory spinal nerve injury.  Clin Orthop 1975;109:15-18.   

 

 

 

 

37.     A collegiate rower reports the sudden onset of right chest pain while rowing.  The athlete states that the pain is worse with deep inspiration and coughing.  Examination reveals localized tenderness over the posterolateral corner of the eighth rib.  What is the most likely diagnosis?

 

1-         Intercostal muscle strain

2-         Rib stress fracture

3-         Pneumothorax

4-         Costochondritis

5-         Empyema

 

PREFERRED RESPONSE: 2

 

DISCUSSION: A rib stress fracture, the most common injury to the thorax in rowing athletes, generally occurs during periods of intense training with a low stroke rate and heavy loads.  It is characterized by the sudden onset of sharp, localized chest pain while rowing.  The fifth through the ninth rib is generally affected, and the diagnosis is best established with a bone scan.  An intercostal muscle strain generally has an insidious onset and may be poorly localized.  Costochondritis affects the anterior costochondral junction.  A pneumothorax and an empyema can cause nonlocalized chest pain but are associated with respiratory distress and systemic physical findings.

 

REFERENCES: Karlson KA: Rib stress fractures in elite rowers.  Am J Sports Med 1998;26:516-520.

Holden DL, Jackson DW: Stress fractures of the ribs in female rowers.  Am J Sports Med 1985;13:342-348.

 

 

 

 

38.     Figures 18a and 18b show the radiographs of a 13-year-old baseball player who sustained a patellar dislocation with an associated lateral femoral condyle fracture.  What ligament is attached to this fragment?

 

1-         Anterior cruciate

2-         Posterior cruciate

3-         Lateral collateral

4-         Oblique popliteal

5-         Intermeniscal

 

PREFERRED RESPONSE: 1

 

DISCUSSION: The anterior cruciate ligament is attached to a portion of the lateral femoral condyle.  The posterior cruciate ligament attaches to the medial femoral condyle.  The lateral collateral and oblique popliteal ligaments attach proximal to this fragment.  The intermeniscal ligament attaches the anterior horns of the menisci.

 

REFERENCES: Jobe CM, Wright M: Anatomy of the knee, in Fu FH, Harner CD, Vince KG (eds): Knee Surgery.  Baltimore, MD, Williams & Wilkins, 1994, pp 1-54.

Moore KL, Dalley AF: Lower limb, in Moore KL, Dalley AF (eds): Clinically Oriented Anatomy, ed 4.  Philadelphia, PA, Lippincott, Williams & Wilkins, 1999, pp 503-664.

 

 

 

 

39.    Which of the following substances does not have androgenic effects?

 

1-         Growth hormone 

2-         Androstenedione 

3-         Creatine 

4-         Dehydroepiandrosterone (DHEA) 

5-         Nandrolone 

 

PREFERRED RESPONSE: 3

 

DISCUSSION: Growth hormone is the most abundant substance produced by the pituitary gland.  Growth hormone has a direct anabolic effect by accelerating the incorporation of amino acids into proteins.  It is becoming an increasingly popular anabolic steroid substitute; however, it is expensive and difficult to obtain.  Androstenedione is an androgen produced by the adrenal glands and gonads.  It acts as a potent anabolic steroid and is converted in the liver directly to testosterone with a resultant increase in levels after administration.  DHEA is a naturally occurring hormone made by the adrenal cortex.  It is converted to androstenedione, which in turn is converted to testosterone.  The beneficial and adverse effects of DHEA can be correlated directly with those of testosterone.  Nandrolone is also a potent anabolic steroid.  It is commonly taken as 19-norandrostenedione and may be more favored because of its potent anabolic effects with less androgenic effects (no conversion to estrogen compounds).  Creatine sales have skyrocketed, and it is a popular nutritional supplement.  There is an expectation that creatine can increase strength and power performance; however, direct anabolic effects have not been demonstrated.  Creatine serves as a substrate for hydrogen ions and contributes to the resynthesis of ATP (adenosine triphosphate) during maximal exercise.  By enhancing ATP   production and buffering local pH in muscle, there may be improved tolerance of anaerobic activities.  Increases in muscle mass may be related to increased perception of improved training ability or an increase in muscle water content.

 

REFERENCES: Silver M: Use of ergogenic aids by athletes.  J Am Acad Orthop Surg 2001;9:61-70.

Blue JG, Lombardo JA: Steroids and steroid-like compounds.  Clin Sports Med 1999;18:667-689.

 

 

 

 

40.    A superior labrum anterior and posterior (SLAP) lesion doubles the strain in which of the following stabilizing structures?

 

1-         Superior glenohumeral ligament

2-         Middle glenohumeral ligament

3-         Inferior glenohumeral ligament

4-         Posterior inferior glenohumeral ligament

5-         Subscapularis

 

PREFERRED RESPONSE: 3

 

DISCUSSION: A superior labrum, when intact, stabilizes the shoulder by increasing its ability to withstand excessive external rotational forces by an additional 32%.  The presence of a SLAP lesion decreases this restraint and increases the strain in the superior band of the inferior glenohumeral ligament by over 100%.

 

REFERENCES: Rodosky MW, Harner CD, Fu FH: The role of the long head of the biceps muscle and superior glenoid labrum in anterior stability of the shoulder.  Am J Sports Med 1994;22:121-130.

Itoi E, Kuelchle DK, Newman SR, Morrey BF, An KN: Stabilizing function of the biceps in stable and unstable shoulders.  J Bone Joint Surg Br 1993;75:546-550.

 

 

41.     What is the principal advantage of surgical repair for the lesion shown in Figure 19?

 

1-         Less risk of repeat rupture

2-         Less pain

3-         Greater motion

4-         Quicker recovery

5-         Greater dorsiflexion strength

 

PREFERRED RESPONSE: 1

 

DISCUSSION: The MRI scan shows a rupture of the Achilles tendon.  The substantiated advantages of repair are less risk of re-rupture and greater plantar flexion strength.  Dorsiflexion strength is not influenced.  Motion, pain, and period of recovery are not specifically improved as a consequence of surgery.

 

REFERENCES: Bhandari M, Guyatt GH, Siddiqui F, et al: Treatment of acute Achilles tendon ruptures: A systematic overview and meta-analysis.  Clin Orthop 2002;400:190-200.

Schepsis AA, Jones HE, Haas AL: Achilles tendon disorders in athletes.  Am J Sports Med 2002;30:287-305.

 

 

 

 

42.    An 18-year-old high school football player sustains a left posterior hip dislocation that is reduced in the emergency department under IV sedation. Postreduction radiographs reveal a concentric reduction with no evidence of fracture or loose bodies within the joint.  What is the most common complication of hip dislocations?

 

1-         Femoral nerve palsy

2-         Sciatic nerve palsy

3-         Recurrent hip dislocation

4-         Osteonecrosis of the femoral head

5-         Immediate chondrolysis of the hip joint

 

PREFERRED RESPONSE: 4

 

DISCUSSION: Traumatic dislocation of the hip in sports injuries is uncommon, and 85% to 92% occur in a posterior direction.  In dislocations without fractures, osteonecrosis is the most common complication occurring in 10% to 20% of patients.  MRI should be performed at 3 months postreduction to rule out osteonecrosis.  Nerve injuries are rare in this setting, and recurrent dislocations are unusual without acetabular fractures.  Chondrolysis has been reported as a rare occurrence.

 

REFERENCES: Anderson K, Strickland S, Warren R: Hip and groin injures in athletes.  Am J Sports Med 2001;29:521-533.

Koval KJ (ed): Orthopaedic Knowledge Update 7.  Rosemont, IL, American Academy of Orthopaedic Surgeons, 2002, pp 407-416.

 

 

43.    A high school athlete reports the sudden onset of low back pain while performing a dead lift.  Examination reveals lumbar paraspinal spasm and a positive straight leg raising test.  Deep tendon reflexes, motor strength, and sensation in the lower extremities are normal.  Radiographic findings are normal.  If symptoms persist for longer than a few weeks, what is the best course of action?

 

1-         Electromyography and nerve conduction velocity studies

2-         MRI

3-         CT

4-         Bone scan

5-         Psychiatric evaluation

 

PREFERRED RESPONSE: 2

 

DISCUSSION: In the adolescent population, a lumbar herniated disk is characterized by a paucity of clinical findings, with a positive straight leg raising test the only consistently positive finding.  This may result in a prolonged period of nonsurgical management that fails to provide relief.  Activities that place a significant shear load on the lumbar spine, such as the dead lift, are associated with an increased risk of central disk herniation.  An adolescent who lifts weights and has a history of back pain that fails to respond to a short period of active rest should undergo MRI evaluation for the diagnosis of a lumber herniated disk.

 

REFERENCES: Epstein JA, Epstein NE, Marc J, et al: Lumbar intervertebral disk herniation in teenage children: Recognition and management of associated anomalies.  Spine 1984;9:427-432.

Hashimoto K, Fujita K, et al: Lumbar disc herniation in children.  J Pediatr Orthop
1990;10:394-396.

 

 

 

 

44.    Examination of a 23-year-old female college basketball player who has had anterior knee pain for the past 3 weeks reveals tenderness and fullness over the inferior patella and proximal patellar tendon.  There is no patellofemoral crepitus, patella apprehension sign, or anterior or posterior instability.  Initial management should include

 

1-         bilateral shoe orthotics to support the medial foot arch.

2-         a very small dose of lidocaine and cortisone injected into the area of pain to assist in diagnosis and treatment.

3-         early lateral retinaculum release with medial soft-tissue tightening.

4-         ice, rest, avoidance of the offending activity, and rehabilitation.

5-         no sports participation for a minimum of 6 weeks.

 

PREFERRED RESPONSE: 4

 

DISCUSSION: The patient has patellar tendinitis (jumper’s knee).  It is a common overuse condition seen in runners, volleyball players, soccer players, and jumpers but can be seen in any activity in which repeated extension of the knee is required.  In the acute setting, the pain is well localized and there is tenderness and sometimes swelling of the tendon.  MRI is recommended for evaluating chronic cases and for surgical planning.  In the acute phases, ice, rest, and avoidance of the offending activity are recommended.  Weakness of the quadriceps and hamstring muscle are thought to contribute to this problem; therefore, stretching and isometric exercise in a limited range of motion are important.  Complete rest and intratendinous injections of steroids are detrimental to tendon physiology.

 

REFERENCES: Stanish WD, Rubinovich RM, Curwin S: Eccentric exercise in chronic tendinitis.  Clin Orthop 1986;208:65-68.

Witvrouw E, Bellemans J, Lysens R, Danneels L, Cambier D: Intrinsic risk factors for the development of patellar tendinitis in an athletic population: A two-year prospective study.  Am J Sports Med 2001;29:190-195.

 

 

 

 

45.     Which of the following findings is likely to be pathologic in a thin, well-conditioned endurance athlete?

 

1-         Left ventricular hypertrophy by voltage on electrocardiography (ECG)

2-         Primary AV block on ECG

3-         II/IV systolic murmur increased with standing and Valsalva maneuver

4-         Nonspecific STT wave changes in the lateral leads on ECG

5-         Resting sinus bradycardia at 40 beats per minute

 

PREFERRED RESPONSE: 3

 

DISCUSSION: Left ventricular hypertrophy by voltage is a nonspecific diagnosis, especially in athletes with an asthenic body habitus.  High vagal tone in endurance athletes may result in first degree or even type I second degree (ie, Wenckebach) AV block in endurance athletes.  High vagal tone results in resting sinus bradycardia in many trained athletes.  A I-II/IV systolic ejection murmur is occasionally found in healthy athletes; however, when the murmur increases in intensity with maneuvers that decrease ventricular filling, such as standing or the Valsalva maneuver, dynamic obstruction that is the result of hypertrophic obstructive cardiomyopathy should be suspected.  Nonspecific STT wave changes in the lateral leads on ECG are not uncommon in highly trained athletes; thus, they are nonspecific for ischemic heart disease.

 

REFERENCES: Pelliccia A, Maron BJ, Culasso F, DiPaolo FM, et al: Clinical significance of abnormal electrocardiographic patterns in trained athletes.  Circulation 2000;102:278-284.

Maron BJ, Thompson PD, Puffer JC, McGrew CA: Cardiovascular preparticipation screening of competitive athletes: A statement for health professionals from the Sudden Death Committee (Clinical Cardiology) and Congenital Cardiac Defects Committee (Cardiovascular Disease in the Young), American Heart Association.  Circulation 1996;94:850-856.

 

 

 

 

46.    Figure 20 shows the radiograph of a 21-year-old college basketball player who jammed his left index finger on the rim.  He reports pain and tenderness over the dorsum of the distal interphalangeal (DIP) joint.  Examination reveals that he is unable to actively extend the DIP joint; however, the skin is intact.  Management should consist of

 

1-         buddy taping to the adjacent digit.

2-         open reduction and surgical fixation of the bony fragment.

3-         excision of the bony fragment and advancement of the terminal extensor mechanism.

4-         splinting of the DIP joint with intermittent removal and range-of-motion exercises to prevent stiffness.

5-         full-time splinting of the DIP joint in slight hyperextension for 6 weeks.

 

PREFERRED RESPONSE: 5

 

DISCUSSION: Mallet fingers without DIP joint subluxation can be treated with extension splinting.  Surgical fixation may be necessary in bony mallet injuries when the joint is subluxated.  Size of the bony fragment, while often correlating with stability, is not always an indication for fixation.  Buddy taping allows motion; therefore, the fragment will not heal in the appropriate position.  Intermittent splinting with range-of-motion exercises also will not allow the fragment to heal in the appropriate position. 

 

REFERENCES: Crawford GP: The molded polyethylene splint for mallet finger deformities. 
J Hand Surg Am 1984;9:231-237.

Wehbe MA, Schneider LH: Mallet fractures.  J Bone Joint Surg Am 1984;66:658-669.

 

 

 

 

47.     With a full-thickness articular cartilage injury, the body’s healing response produces cartilage mainly composed of what type of collagen?

 

1-         I

2-         II

3-         III

4-         IV

5-         X

 

PREFERRED RESPONSE: 1

 

DISCUSSION: With a full-thickness articular cartilage injury, a healing response is initiated with hematoma, stem cell migration, and vascular ingrowth.  This response produces type I collagen and resultant fibrous cartilage rather than desired hyaline cartilage as produced by chondrocytes.  This repair cartilage has diminished resiliency, stiffness, poor wear characteristics, and the predilection for arthritis.  Type I collagen is also found in the annulus of intervertebral disks, tendon, bone, meniscus, and skin.  Type II is found in articular cartilage and nucleus pulposus of intervertebral disks.  Type III is found in skin and blood vessels, type IV is found in basement membranes, and type X is found in the calcified layer of cartilage. 

 

REFERENCES: Arendt EA (ed): Orthopaedic Knowledge Update: Sports Medicine 2.  Rosemont, IL, American Academy of Orthopaedic Surgeons, 1999, pp 19-28.

Brinker M: Basic Science in Review of Orthopaedics, ed 3.  Philadelphia, PA,
WB Saunders, 2000.

 

 

 

 

48.     A relative contraindication for anteromedial tibial tubercle transfer for patellar instability is arthrosis in what portion of the patella?

 

1-         Lateral

2-         Lateral and inferior

3-         Central

4-         Medial

5-         Medial and proximal

 

PREFERRED RESPONSE: 5

 

DISCUSSION: Anteromedial displacement of the tibial tubercle unloads the distal and lateral facets of the patella and shifts the forces to the proximal and medial facets.  Therefore, if findings indicate arthrosis predominately in the medial and proximal areas of the patella, this is considered a relative contraindication because it may accentuate arthritic symptoms.

 

REFERENCES: Fulkerson JP: Anteromedialization of the tibial tuberosity for patellofemoral malalignment.  Clin Orthop 1983;177:176-181.

Bellemans J, Cauwenberghs F, Witvrouw E, et al: Anteromedial tibial tubercle transfer in patients with chronic anterior knee pain and a subluxation-type patellar malalignment.  Am J Sports Med 1997;25:375-381.

Kuroda R, Kambic H, Valdevit A, et al: Articular cartilage contact pressure after tibial tuberosity transfer: A cadaveric study.  Am J Sports Med 2001;29:403-409.

 

 

 

49.    An 18-year-old lacrosse player sustained a hamstring pull during a game.  Examination the next day reveals ecchymosis through the posterior thigh and a palpable defect in the hamstring musculature in the middle third of the thigh.  What is the most likely site of anatomic injury?

 

1-         Rupture of the biceps femoris at the myotendinous junction

2-         Rupture of the biceps femoris muscle belly

3-         Avulsion of the common hamstring origin from the ischium

4-         Complete rupture of the semimembranosus muscle belly

5-         Complete tear of all hamstring muscles

 

PREFERRED RESPONSE: 1

 

DISCUSSION: Hamstring strains are common in athletes.  Basic science research and clinical data indicate that the majority of these injuries occur at the myotendinous junction, not within the muscle belly.  Avulsion of hamstring origin from the ischial tuberosity does occur but is less common.  Complete tearing of all hamstring muscles is unlikely to occur.

 

REFERENCES: Griffin LY (ed): Orthopaedic Knowledge Update: Sports Medicine.  Rosemont, IL, American Academy of Orthopaedic Surgeons, 1994, pp 17-33.

Clanton TO, Coupe KJ: Hamstring strains in athletes: Diagnosis and treatment.  J Am Acad Orthop Surg 1998;6:237-248.

 

 

 

 

50.    Figures 21a through 21c show the MRI scans of a 21-year-old football player who sustained a valgus knee injury while changing direction.  Examination reveals swelling and tenderness along the medial aspect of the knee.  There is a positive Lachman test, 3+ valgus laxity at 30 degrees, and 1+ valgus laxity at 0 degrees extension.  The anterior drawer test is increased with the tibia in external rotation.  The increase in the anterior drawer test with the tibia in external rotation is most likely the result of

 

1-         an occult fracture of the tibial plateau.

2-         a tear of the medial collateral ligament and the posteromedial capsule.

3-         a tear of the posterior cruciate ligament.

4-         a tear of the anterior and posterior cruciate ligaments.

5-         a tear of the anterior cruciate and medial collateral ligaments and the posteromedial capsule.

 

PREFERRED RESPONSE: 5

 

DISCUSSION: The injury mechanism involves a valgus load applied to the knee with the foot in external rotation.  The primary stabilizer to valgus laxity is the medial collateral ligament.  The secondary restraints to valgus rotation are the cruciate ligaments.  Examination indicates disruption of the medial collateral and anterior cruciate ligaments.  Valgus opening in extension should also arouse suspicion for an injury to the posterior cruciate ligament; however, in this patient, the valgus opening in extension is mild.  The slight opening in extension and the increased anterior drawer, especially with external rotation, indicates disruption of the posteromedial capsule and posterior oblique ligament.  Figure 21a shows complete disruption of the superficial and deep medial collateral ligaments involving the meniscofemoral ligament.  Figure 21b shows a more posterior coronal section with a torn posterior oblique ligament.  Figure 21c shows disruption of the anterior cruciate ligament, while the posterior cruciate ligament at the tibial insertion appears with a homogenous normal signal.

 

REFERENCES: Warren LA, Marshall JL, Girgis F: The prime static stabilizer of the medial side of the knee.  J Bone Joint Surg Am 1974;56:665-674.

Indelicato PA: Injury to the medial capsuloligamentous complex of the knee, in Feagin J (ed): The Crucial Ligaments, ed 2. 1994, pp 351-360.

 

 

 

51.     A 32-year-old amateur bowler has progressive pain in the lateral aspect of the proximal forearm and elbow.  Nonsurgical management consisting of a tennis elbow brace, nonsteroidal anti-inflammatory drugs, and activity modification has failed to provide relief.  Examination reveals tenderness in the lateral aspect of the proximal forearm and exacerbation of symptoms with resisted finger extension.  Radiographs of the elbow reveal no abnormalities.  Which of the following studies will aid in diagnosis?

 

1-         MRI of the elbow and forearm

2-         Bone scan

3-         Electrodiagnostic studies

4-         Radial tunnel injection

5-         Radiographs of the wrist

 

PREFERRED RESPONSE: 4

 

DISCUSSION: It is often difficult to accurately discern between lateral epicondylitis and radial tunnel syndrome.  Neither MRI nor a bone scan is likely to reveal abnormalities.  Electrodiagnostic studies are often inconclusive, and radial tunnel syndrome often presents without motor weakness.  The symptoms of radial tunnel syndrome are expected to improve with an injection of lidocaine into the radial tunnel; therefore, this is the test of choice in this clinical scenario.  Radiographs of the wrist will not assist in making the diagnosis.

 

REFERENCES: Eversmann WW Jr: Entrapment and compression neuropathies, in Green DP (ed): Operative Hand Surgery, ed 3.  New York, NY, Churchill Livingston, 1993, pp 1341-1385.

Sarhadi NS, Korday SN, Bainbridge LC: Radial tunnel syndrome: Diagnosis and management.  J Hand Surg Br 1998;23:617-619.

 

 

 

 

52.     What is the most common complication associated with scalene regional anesthesia for shoulder procedures?

 

1-         Cardiovascular collapse

2-         Block failure

3-         Seizure secondary to intravascular injection

4-         Phrenic nerve injury

5-         Laryngeal nerve injury

 

PREFERRED RESPONSE: 2

 

DISCUSSION: Failure of the scalene block, necessitating general anesthesia or the immediate administration of narcotic medications, is the most common complication, occurring in 3% to 18% of patients. Cardiac arrest or cardiovascular collapse has been reported in anecdotal occurrences.  Seizure that is the result of intravascular injection of local anesthetic is a rare complication, with an incidence reported of 0% to 6%.  Neurologic complications, including laryngeal and phrenic nerve injuries, are rare although parathesias lasting up to 2 weeks have been reported in up to 3% of patients.

 

REFERENCES: Weber SC, Jain R: Scalene regional anesthesia for shoulder surgery in a community setting: An assessment of risk.  J Bone Joint Surg Am 2002;84:775-779.

Conn RA, Colfield RH, Byer DE, Lindstromberg JW: Interscalene block anesthesia for shoulder surgery.  Clin Orthop 1987;216:94-98.

 

 

 

 

53.     Figure 22 shows the MRI scan of a 20-year-old female basketball player who has pain over the anterior knee that interferes with her performance.  Examination reveals phase III Blazina patellar tendinosis.  Management should consist of

 

1-         local modalities including iontophoresis.

2-         quadriceps and iliotibial band stretching exercises.

3-         progressive eccentric strengthening exercises.

4-         a patellar tendon strap.

5-         excision of the abnormal area.

 

PREFERRED RESPONSE: 5

 

DISCUSSION: Excision of the affected mucoid degenerative area is considered appropriate management in the Blazina classification system.  A finding of phase III indicates persistent pain with or without activities, as well as deterioration of performance.  With the appearance of the mucoid degeneration and the vigorous activity level of the intercollegiate basketball player, it is unlikely that nonsurgical management will provide adequate relief.  When excising the affected degenerative area, care must be taken to retain normal tendon fibers.  The defect in the patellar tendon is closed with absorbable sutures, as is the paratenon.  Postoperative rehabilitation involves initial mobilization extension, with progressive range-of-motion and mobilization exercises as tolerated and weight bearing as tolerated.  Open chain and isokinetic exercises are delayed until full range of motion and mobility is obtained, generally within 4 weeks.  A return to activities is achieved by 80% to 90% of athletes, although there may be occasional activity-related aching for 4 to 6 months after surgery.

 

REFERENCES: Blazina ME, et al: Jumper’s knee.  Orthop Clin North Am 1973;4:665.

Kelly DW, Carter VS, Jobe FW, Kerlan RK: Patellar and quadriceps tendon ruptures: Jumper’s knee.  Am J Sports Med 1984;12:375-380.

Krums PE, Ryder B: Operative treatment of patella tendon disorders.  Operative Techniques Sports Med 1994;2:303.

Rows J, et al: Patella tendinitis (jumper’s knees).  Am J Sports Med 1978;6:362.

 

 

 

 

54.     When comparing the failure load of an evenly tensioned four-stranded hamstring tendon anterior cruciate ligament autograft to a 10-mm bone-patellar tendon-bone autograft, the hamstring graft will fail at a tension

 

1-         equal to the bone-patellar tendon-bone graft.

2-         one half the failure load of the bone-patellar tendon-bone graft.

3-         one quarter the failure load of the bone-patellar tendon-bone graft.

4-         approximately two times the failure load of the bone-patellar tendon-bone graft.

5-         four times the failure load of the bone-patellar tendon-bone graft.

 

PREFERRED RESPONSE: 4

 

DISCUSSION: The failure load of an evenly tensioned four-stranded hamstring tendon autograft has been reported to be 4,500 Newtons.  The failure load of a 10-mm patellar tendon autograft has been estimated at 2,600 Newtons.  The intact anterior cruciate ligament failure load has been calculated at 1,725 Newtons.

 

REFERENCES: Corry IS, Webb JM, Clingeleffer AJ, Pinczewski LA: Arthroscopic reconstruction of the anterior cruciate ligament: A comparison of patellar tendon autograft and four-strand hamstring tendon autograft.  Am J Sports Med 1999;27:448-454.

Hamner DL, Brown CH Jr, Steiner ME, et al: Hamstring tendon grafts for reconstruction of the anterior cruciate ligament: Biomechanical evaluation of the use of multiple strands and tensioning techniques.  J Bone Joint Surg Am 1999;81:549-557.

Noyes FR, Butler DL, Grood ES, et al: Biomechanical analysis of human ligament grafts used in knee-ligament repairs and reconstructions.  J Bone Joint Surg Am 1984;66:344-352.

 

 

 

 

55.     What pathology is most likely to result in failure of an arthroscopic Bankart repair?

 

1-         A 25% or greater anterior-inferior glenoid rim defect

2-         Nonengaging moderate Hill-Sachs defect

3-         Associated type IV superior labrum anterior and posterior (SLAP) lesion  

4-         Absence of an intact cartilaginous labrum

5-         Atttenuated anterior-inferior capsule and glenohumeral ligament complex

 

PREFERRED RESPONSE: 1

 

DISCUSSION: Recent studies have documented that an arthroscopic Bankart repair performed with good technique can produce success rates similar to an open repair.  However, the results of an arthroscopic repair deteriorate significantly if there is a 25% or greater anterior-inferior glenoid rim defect (inverted pear configuration) or an engaging Hill-Sachs lesion in which the humeral head defect keys onto the glenoid rim in abduction and external rotation.  If either of these entities exist or there is multidirectional instability with pathologic hyperextensible tissue laxity, an open repair is recommended.  An associated SLAP lesion would not significantly affect the result of the Bankart procedure.  Not infrequently, the anterior glenoid labrum is partially or completely disrupted and, in itself, is not a contraindication to arthroscopic Bankart repair.  In almost all patients with predominantly unidirectional instability, some degree of capsular/anterior-inferior glenohumeral ligament attenuation is present and can be addressed during the arthroscopic repair.

 

REFERENCES: Burkhart SS, De Beer JF: Traumatic glenohumeral bone defects and their relationship to failure of arthroscopic Bankart repairs: Significance of the inverted-pear glenoid and the humeral engaging Hill-Sachs lesion.  Arthroscopy 2000;16:677-694. 

Cole BJ, Romeo AA: Arthroscopic shoulder stabilization with suture anchors: Technique, technology, and pitfalls.  Clin Orthop 2001;390:17-30.

 

 

 

 

56.     A 37-year-old man has had isolated chronic knee swelling for the past 6 months.  He denies any history of specific trauma.  Examination reveals a large effusion with a stable knee, but the remainder of the examination is normal.  Plain radiographs are unremarkable.  An MRI scan reveals a large effusion without meniscal injury.  An arthroscopic image of the suprapatellar pouch is shown in Figure 23.  What is the most likely diagnosis?

 

1-         Septic arthritis

2-         Chondromalacia of the medial femoral condyle

3-         Synovial cell sarcoma

4-         Rheumatoid arthritis

5-         Pigmented villonodular synovitis (PVNS)

 

PREFERRED RESPONSE: 5

 

DISCUSSION: The history and physical examination are consistent with a monoarticular joint condition but not typical of joint sepsis.  The arthroscopic appearance of brownish proliferative synovium is typical of PVNS.  PVNS is a monoarticular synovial disease of unknown etiology and is treated with total synovectomy.  The proliferative synovitis is not consistent with chondromalacia.  Synovial cell sarcoma is an extracapsular disease.  Rheumatoid arthritis typically is polyarticular, and the synovial appearance is not associated with hemosiderin deposition.

 

REFERENCES: Flandry FC, Hughston JC, Jacobson KE, Barrack RL, McCann SB, Kurtz DM: Surgical treatment of diffuse pigmented villonodular synovitis of the knee.  Clin Orthop 1994;300:183-192.

Zvijac JE, Lau AC, Hechtman KS, Uribe JW, Tjin-A-Tsoi EW: Arthroscopic treatment of pigmented villonodular synovitis of the knee.  Arthroscopy 1999;15:613-617.

 

 

 

 

57.     A 35-year-old recreational basketball player reports shoulder pain following a sprawl for a rebound.  While examination reveals that he can actively elevate the arm with pain, a subacromial injection fails to provide relief.  An MRI scan reveals medial subluxation of the long head of the biceps.  Which of the following structures most likely has also been injured?

 

1-         Inferior glenohumeral ligament

2-         Middle glenohumeral ligament

3-         Superior labrum

4-         Subscapularis tendon

5-         Supraspinatus tendon

 

PREFERRED RESPONSE: 4

 

DISCUSSION: Subscapularis tears can be associated with disruption of the transverse ligament supporting the biceps.  The remaining aspects of the rotator cuff, superior labrum, and capsule can be intact with this injury.

 

REFERENCES: Petersson CJ: Spontaneous medial dislocation of the tendon of the long biceps brachii.  Clin Orthop 1986;211:224-227.

Gerber C, Sebesta A: Impingement of the deep surface of the subscapularis tendon and the reflection pulley on the anterosuperior glenoid rim: A preliminary report.  J Shoulder Elbow Surg 2000;9:483-490.

 

 

 

 

58.     An 18-year-old hockey player sustains an acute anterior shoulder dislocation that requires manual reduction.  At arthroscopy, the lesion shown in Figure 24 will be observed in what percent of patients?

 

1-         20% to 30%

2-         35% to 45%

3-         50% to 60%

4-         80% to 95%

5-         100%

 

PREFERRED RESPONSE: 4

 

DISCUSSION: The clinical photograph shows an acute capsulolabral avulsion from the anterior glenoid, also referred to as a Perthes-Bankart lesion.  In patients who sustain an acute dislocation that requires a manual reduction, this pathologic lesion is observed with high frequency.  In several research studies, it has been visualized in 80% to 95% of patients at arthroscopy.

 

REFERENCES: Taylor DC, Arciero RA: Pathologic changes associated with shoulder dislocations: Arthroscopic and physical examination findings in first-time, traumatic anterior dislocations.  Am J Sports Med 1997;25:306-311.

Baker CL, Uribe JW, Whitman C: Arthroscopic evaluation of acute initial anterior shoulder dislocations.  Am J Sports Med 1990;18:25-28.

 

 

 

 

59.     A 22-year-old competitive volleyball player has shoulder pain, and rest and a cortisone injection have failed to provide relief.  Examination reveals atrophy along the posterior scapula, but an MRI scan does not reveal a rotator cuff tear or labral cyst.  What is the most likely cause for the shoulder weakness?

 

1-         Biceps tear

2-         Bankart lesion

3-         Teres minor avulsion

4-         Suprascapular nerve injury

5-         Superior labrum anterior and posterior tear

 

PREFERRED RESPONSE: 4

 

DISCUSSION: Repetitive overhead slams and serves may produce a traction injury to the distal branch of the suprascapular nerve.  Bankart, biceps, and superior labrum anterior and posterior injuries can occur but usually do not produce visible atrophy.  Muscle avulsion is uncommon.

 

REFERENCES: Ferretti A, Cerullo G, Russo G: Suprascapular neuropathy in volleyball players.  J Bone Joint Surg Am 1987;69:260-263.

Bigliani LU, Dalsey RM, McCann PD, April EW: An anatomical study of the suprascapular nerve.  Arthroscopy 1990;6:301-305.

 

 

 

 

60.    An 11-year-old female gymnast has had gradually increasing right wrist pain for the past 6 months.  Examination reveals normal range of motion and strength.  Moderate tenderness is present over the distal radius.  AP radiographs will most likely show

 

1-         overgrowth of the distal radial epiphysis.   

2-         premature closure of the distal radial physis.

3-         premature closure of the distal ulnar physis.

4-         a Salter-Harris type I fracture of the distal radius with a volar slip of the epiphysis.

5-         a Salter-Harris type I fracture of the distal radius with a dorsal slip of
the epiphysis. 

 

PREFERRED RESPONSE: 2

 

DISCUSSION: Distal radial physeal stress syndrome has been reported in up to 25% of nonelite gymnasts showing premature closure of the distal radial physis and distal ulnar overgrowth, producing positive ulnar variance.  The diagnosis should be suspected when there is tenderness at the distal radial physis in a young gymnast.  The pathology is thought to be the result of repetitive compressive stresses caused by upper extremity weight-bearing forces.  The recommended treatment is 3 to 6 months of rest.  Salter-Harris fractures with a distal radial epiphyseal slip are unlikely, especially in the absence of a specific traumatic event.

 

REFERENCES: Mandelbaum BR, Bartolozzi AR, Davis CA, Teurlings L, Bragonier B: Wrist pain syndrome in the gymnast: Pathogenetic, diagnostic, and therapeutic consideration.  Am J Sports Med 1989;17:305-317.

Roy S, Caine D, Singer KM: Stress changes of the distal radial epiphysis in young gymnasts: A report of twenty-one cases and a review of the literature.  Am J Sports Med 1985;13:301-308. 

 

 

 

 

61.     A 22-year-old wrestler who underwent an open anterior shoulder reconstruction to repair a dislocated shoulder 6 months ago now reports shoulder pain after attempting a takedown.  Examination reveals external rotation that is 15 degrees greater than the contralateral side.  He has pain associated with abduction and external rotation but no apprehension.  Which of the following tests would most likely reveal positive findings?

 

1-         Impingement injection test

2-         Lift-off test

3-         Weakness with “empty-can” abduction test

4-         Load-and-sift maneuver

5-         MRI with contrast

 

PREFERRED RESPONSE: 2

 

DISCUSSION: Postoperative subscapularis detachment can be identified with a positive lift-off test that reveals weakness in internal rotation.  This complication does not necessarily compromise the anterior capsule repair.  The load-and-sift maneuver and articular contrast studies may be normal.  Supraspinatus tests for impingement and weakness should be negative.

 

REFERENCES: Gerber C, Krushell RJ: Isolated ruptures of the tendon of the subscapularis muscle: Clinical fractures in 16 cases.  J Bone Joint Surg Br 1991;73:389-394.

Hawkins RJ, Bokor DJ: Clinical evaluation of the shoulder, in Rockwood CA, Matsen FA III (eds): The Shoulder.  Philadelphia, PA, WB Saunders, 1990, pp 149-177.

 

 

 

 

62.    Figures 25a and 25b show the clinical photographs of a 19-year-old baseball outfielder who has shoulder pain after sliding headfirst into second base.  He reports pain while batting, sliding, and catching.  Examination reveals a posterior prominence during midranges of forward elevation, which then disappears with a palpable clunk during terminal elevation and abduction.  What is the most likely diagnosis?

 

1-         Superior labrum anterior and posterior (SLAP) lesion

2-         Bankart lesion

3-         Rotator cuff interval tear

4-         Rotator cuff tendinitis

5-         Posterior glenohumeral subluxation

 

PREFERRED RESPONSE: 5

 

DISCUSSION: A headfirst slide with the arm extended can injure the posterior shoulder.  Winging of the scapula is dynamic and is considered a compensatory effort to prevent subluxation; it is not related to nerve injury.  Posterior glenohumeral subluxation can be present during the initiation of a bat swing.  Rotator cuff function, interval tears, and superior labrum tears can be painful but do not produce winging.

 

REFERENCES: Kuhn JE, Plancher KD, Hawkins RJ: Scapular winging.  J Am Acad Orthop Surg 1995;3:319-325.

Fiddian NJ, Kling RJ: The winged scapula.  Clin Orthop 1984;185:228-236.

 

 

 

 

63.    A soccer player who sustained a twisting injury to the right ankle while making a cut is unable to bear weight and has diffuse tenderness over the anterior and lateral aspects of the ankle.  Examination also shows a positive squeeze test.  Plain radiographs and a stress radiograph are shown in Figures 26a through 26c.  Radiographs of the leg and knee are normal.  What is the most appropriate management?

 

1-         Short leg non-weight-bearing cast for 6 weeks

2-         Air-stirrup splint and limited activity in 3 to 6 weeks

3-         Air-stirrup splint and resumption of activities as tolerated

4-         Immediate repair of the peroneal retinaculum

5-         Immediate reduction and placement of a syndesmotic screw

 

PREFERRED RESPONSE: 5

 

DISCUSSION: The mechanism of injury, physical examination, and radiographs indicate a “high” ankle sprain with disruption of the distal tibiofibular ligaments and interosseous membrane.  These injuries typically involve pronation and external rotation forces.  In addition, recovery is significantly delayed, often requiring 6 to 8 weeks to heal.  Radiographs obtained months after recovery often show calcification within the distal syndesmosis, which is not typically symptomatic.  This patient has gross instability, resulting in a high incidence of chronic diastasis and subluxation leading to impaired function.  Treatment should consist of reduction and stabilization with a transsyndesmotic screw because this injury demonstrates a widened syndesmosis.

 

REFERENCES: Boytim MJ, Fisher DA, Neumann L: Syndesmotic ankle sprains.  Am J Sports Med 1991;19:294-298.

Miller CD, Shelton WR, Barrett GR, et al: Deltoid and syndesmosis ligament injury of the ankle without fracture.  Am J Sports Med 1995;23:746-750.

 

 

 

 

64.    When compared with the normal anterior cruciate ligament (ACL), placement of an anterior cruciate ligament graft in the over-the-top position on the femoral side has what effect on its function?

 

1-         Lax in flexion and tight in extension

2-         Lax in flexion and lax in extension

3-         Tight in flexion and lax in extension

4-         Tight in flexion and tight in extension

5-         Remains isometric

 

PREFERRED RESPONSE: 1

 

DISCUSSION: The placement of ACL graft with respect to its femoral and tibial attachments has a significant effect on its function.  Evidence has shown that if the graft is placed in the over-the-top position, the graft will become lax in flexion and more taut with extension.  Conversely, if the graft is placed too anterior on the femoral side, it will tighten in flexion and become lax in extension.

 

REFERENCES: Azar FM: Revision anterior cruciate ligament reconstruction.  Instr Course Lect 2002;51:335-342. 

Draganich LF, Hsieh YF, Sherwin SH, et al: Intra-articular anterior cruciate ligament graft placement on the average most isometric line on the femur: Does it reproducibly restore knee kinematics?  Am J Sports Med 1999;27:329-334.

Bylski-Austrow DI, Grood ES, Hetzy MS, et al: Anterior cruciate ligament replacements: A mechanical study of femoral attachment location, flexion angle at tensioning, and initial tensioning.  J Orthop Res 1990;8:522-531.

 

 

 

 

65.     An 11-year-old boy has right shoulder pain and has been unwilling to use the arm after throwing a baseball in a Little League game 3 weeks ago.  Examination reveals upper arm and shoulder tenderness with swelling.  A radiograph and MRI scan are shown in Figures 27a and 27b.  Management should consist of

 

1-         irrigation, debridement, and IV antibiotics.

2-         curettage and bone grafting.

3-         preoperative chemotherapy followed by wide excision.

4-         observation.

5-         aspiration and injection with methylprednisolone.

 

PREFERRED RESPONSE: 5

 

DISCUSSION: The radiograph is consistent with a unicameral (simple) bone cyst.  The MRI scan reveals that the cyst is juxtaposed to the physis and therefore can be classified as active (latent cysts are more than 1 cm away from the physis).  Active cysts are treated with aspiration and steroid injection, although repeated injections may be necessary.  Curettage and bone grafting results in more reliable healing but may lead to growth arrest in active cysts.

 

REFERENCES: Iannotti JP, Williams GR: Disorders of the Shoulder: Diagnosis and Management, ed 1.  Philadelphia, PA, Lippincott Williams & Wilkins, 1999, pp 945-946.

Malawer MM: Tumors of the shoulder girdle: Techniques of resection and description of surgical classification.  Orthop Clin North Am 1991;22:7-35.

 

 

 

 

66.    A 37-year-old recreational tennis player undergoes surgery for tennis elbow.  Following surgery, she describes clicking and popping on the lateral aspect of the elbow.  A lateral pivot shift test is positive.  What is the most likely cause of her symptoms?

 

1-         Injury to the anterior band of the medial collateral ligament

2-         Injury to the radial nerve

3-         Injury to the lateral ulnar collateral ligament

4-         Injury to the lateral radial collateral ligament

5-         Excessive dissection of the extensor carpi radialis brevis origin

 

PREFERRED RESPONSE: 3

 

DISCUSSION: The patient has a posterolateral rotatory instability (PLRI) of the elbow that is most likely the result of iatrogenic injury to the lateral ulnar collateral ligament, the main ligament implicated in PLRI.  The anterior band of the medial collateral ligament is implicated in valgus instability.  Injury to the radial nerve is unlikely, and the lateral radial collateral ligament makes less of a contribution to elbow stability than does the ulnar component.  While the origin of the extensor carpi radialis brevis may contribute to elbow stability, it is not as important a stabilizer as the lateral ulnohumeral ligament.

 

REFERENCES: O’Driscoll SW, Morrey BF: Surgical reconstruction of the lateral collateral ligament, in Morrey BF (ed): The Elbow.  Philadelphia, PA, Lippincott, Williams and Wilkins, 1994, pp 169-182.

O’Driscoll SW, Bell DF, Morrey BF: Posterolateral rotatory instability of the elbow.  J Bone Joint Surg Am 1991;73:440-446.

 

 

 

 

67.     An 18-year-old football halfback reports that he had immediate right knee pain after being tackled 1 week ago.  Examination now reveals moderate tenderness over the proximal medial tibia and lateral joint and normal cruciate stability.  In evaluating the integrity of the posterolateral knee structures, what is the most reliable examination finding?

 

1-         Excessive varus laxity at 30 degrees of flexion

2-         Reverse pivot shift  

3-         Posterolateral drawer laxity at 90 degrees of flexion

4-         Asymmetric tibial external rotation at 30 degrees of flexion

5-         Positive external rotation/recurvatum test

 

PREFERRED RESPONSE: 4

 

DISCUSSION: The most reliable test for a relatively isolated posterolateral complex (PLC) injury is the asymmetric tibial external rotation or “dial test.”  It can be performed with the patient prone or supine.  When greater than 10 degrees of external rotation at 30 degrees of flexion is present when compared with the opposite knee, it indicates significant damage to the posterolateral structures.  Asymmetric external rotation, which is also present at 90 degrees of flexion, indicates injury to the posterior cruciate ligament (PCL) as well.  Varus laxity may indicate significant damage to both the PLC and PCL.  Approximately 35% of the normal population may have a reverse pivot shift when examined under anesthesia; therefore, it is considered a less specific test.  The external rotation/recurvatum and posterolateral drawer tests are adjunctive in assessing isolated posterolateral laxity but are not thought to be as reliable.

 

REFERENCES: Veltri DM, Warren RF: Isolated and combined posterior cruciate injuries.  J Am Acad Orthop Surg 1993;1:67-75.

Koval KJ (ed): Orthopaedic Knowledge Update 7.  Rosemont IL, American Academy of Orthopaedic Surgeons, 2002, pp 489-511.

 

 

 

 

68.     Figures 28a through 28d show the radiographs and MRI scans of a 20-year-old basketball player who sustained an inversion injury to his right ankle.  Management should
consist of

 

1-         open reduction and internal fixation.

2-         a short leg cast for 6 weeks.

3-         ankle arthroscopy, removal of the fragment, and drilling of the base of the lesion.

4-         ankle arthroscopy and internal fixation.

5-         functional ankle rehabilitation that emphasizes range of motion, peroneal strengthening, and proprioceptive training.

 

PREFERRED RESPONSE: 3

 

DISCUSSION: Osteochondral fractures involving the talar dome have been classified based on radiographic and MRI findings.  A nondisplaced and incomplete fracture may be treated effectively with a short leg cast and no weight bearing for 6 weeks.  This patient has a complete, separated, and displaced osteochondral fragment involving the midlateral talar dome that will most likely cause pain, mechanical symptoms, and effusion if treated nonsurgically.  In addition, there is very little bone remaining on the fragment, making the likelihood of healing with open reduction and internal fixation problematic.  The treatment of choice includes arthroscopy, removal of the loose fragment, curettage or drilling of the base, and a rehabilitation program that emphasizes peroneal strengthening, range of motion, and proprioceptive training.

 

REFERENCES: Lutter LD, Mizel MS, Pfeffer GB (eds): Orthopaedic Knowledge Update: Foot and Ankle.  Rosemont, IL, American Academy of Orthopaedic Surgeons, 1994, pp 205-226.

Baker CL, Morales RW: Arthroscopic treatment of tranchondral talar dome fractures: A long-term follow-up study.  Arthroscopy 1999;15:197-202.

 

 

 

 

69.    A 19-year-old rugby player has severe knee pain after being injured in a game 2 weeks ago.  Examination reveals a knee effusion, limited motion, and increased 3+ Lachman’s test and anterior drawer.  There is also increased external rotation at 30 degrees of knee flexion when the patient is placed in the prone position.  Based on these findings, which of the following actions would most likely increase the risk of anterior cruciate ligament (ACL) reconstruction failure?

 

1-         Inadvertent rotation of the graft 90 degrees internally prior to its final fixation

2-         Lack of full knee extension at the time of surgery

3-         Persistent posterolateral corner injury

4-         Leaving 1 to 2 mm of bone posterior to the femoral tunnel at the time of the ACL reconstruction

5-         Placing the tibial tunnel within the ACL footprint

 

PREFERRED RESPONSE: 3

 

DISCUSSION: The patient has a combined ACL and posterolateral corner injury.  Failure to diagnose and treat an injury of the posterolateral corner in a patient who has a tear of the anterior or posterior cruciate ligament can result in failure of the reconstructed cruciate ligament.  The tibial external rotation test is best performed with the patient in the prone position.  A 10-degree side-to-side difference of external rotation at 30 degrees of knee flexion indicates injury to the posterolateral corner.  Acute grade III isolated or combined injuries of the posterolateral corner are best treated early by direct repair or by augmentation or reconstruction of all injured ligaments.  Postoperative arthrofibrosis after an ACL reconstruction has been observed with preoperative deficiencies of knee motion.

 

REFERENCES: Veltri DM, Warren RF: Posterolateral instability of the knee.  J Bone Joint Surg Am 1994;76:460-472.

Covey DC: Injuries of the posterolateral corner of the knee.  J Bone Joint Surg Am
2001;83:106-118.

 

 

 

 

70.     The primary function of structure “A” in Figure 29 is to limit

 

1-         posterior tibial displacement at 90 degrees of flexion.

2-         varus knee laxity at 30 degrees of flexion.

3-         varus knee laxity at 0 degrees of flexion.

4-         anterolateral rotation of the tibia on the femur.

5-         posterolateral rotation of the tibia on the femur.

 

PREFERRED RESPONSE: 5

 

DISCUSSION: The primary function of the popliteofibular ligament is to resist posterolateral rotation of the tibia on the femur, although it also secondarily resists varus angulation and posterior displacement of the tibia on the femur.  The posterior cruciate ligament resists posterior tibial displacement, especially at 90 degrees of flexion.  The lateral collateral ligament primarily resists varus displacement at 30 degrees of flexion but also resists posterolateral rotatory displacement with flexion that is less than approximately 50 degrees.  The anterior and posterior cruciate ligaments resist varus displacement (along with the lateral collateral ligament) at 0 degrees of flexion.  The anterior cruciate ligament primarily resists anterolateral displacement of the tibia on the femur.

 

REFERENCES: Sugita T, Amis AA: Anatomic and biomechanical study of the lateral collateral and popliteofibular ligaments.  Am J Sports Med 2001;29:466-472.

Veltri DM, Deng XH, Torzilla PA, et al: The role of the cruciate and posterolateral ligaments in stability of the knee: A biomechanical study.  Am J Sports Med  1995;23:436-443.

 

 

 

 

71.     While lifting weights, a patient feels a pop in his arm.  He has the deformity shown in Figure 30.  If left untreated, the patient will have the greatest deficiency in  

 

1-         shoulder flexion.

2-         elbow flexion.

3-         forearm pronation.

4-         forearm supination.

5-         wrist flexion.

 

PREFERRED RESPONSE: 4

 

DISCUSSION: The patient has a distal biceps rupture.  While the distal biceps contributes to elbow flexion, its main function is forearm supination.

 

REFERENCES: Baker BE, Bierwagen D: Rupture of the distal tendon of the biceps brachii: Operative versus non-operative treatment.  J Bone Joint Surg Am 1985;67:414-417.

D’Arco P, Sitler M, Kelly J, et al: Clinical, functional, and radiographic assessments of the conventional and modified Boyd-Anderson surgical procedures for repair of distal biceps tendon ruptures.  Am J Sports Med 1998;26:254-261. 

Pearl ML, Bessos K, Wong K: Strength deficits related to distal biceps tendon rupture and repair: A case report.  Am J Sports Med 1998;26:295-296.

 

 

 

 

72.     Myositis ossificans is a recognized complication of contusion to the quadriceps muscle.  During early rehabilitation, this condition is most likely to be exacerbated by

 

1-         electrical stimulation.

2-         iontophoresis.

3-         isometric exercise.

4-         ice/heat contrast.

5-         passive stretching.

 

PREFERRED RESPONSE: 5

 

DISCUSSION: Passive stretching is contraindicated during rehabilitation as it may potentiate the severity of the myositis ossificans.  Electrical stimulation, iontophoresis, isometric exercise, and ice/heat contrast are not known to exacerbate this process.

 

REFERENCES: Brunet ME, Hontas RB: The thigh, in DeLee JC, Drez D (eds): Orthopaedic Sports Medicine.  Philadelphia, PA, WB Saunders, 1994, pp 1086-1112.

Cushner FD, Morwessel RM: Myositis ossificans traumatica.  Orthop Rev 1992;21:1319-1326.

 

 

 

 

73.     Which of the following symptoms are most commonly associated with piriformis syndrome?

 

1-         Posterior hip and radiating extremity pain

2-         Posterior hip pain and weakness in hip extension

3-         Pain with resisted hip abduction and hypesthesia of the lateral thigh

4-         Painful hip flexion and radiating medial thigh pain

5-         Weakness with hip internal rotation and hypesthesia of the perineum

 

PREFERRED RESPONSE: 1

 

DISCUSSION: Piriformis syndrome is best characterized by localized posterior hip pain and radicular symptoms in the sciatic distribution because of compression of the piriformis muscle on the sciatic nerve.  Weakness in hip extension is not a characteristic finding, nor is pain with hip abduction or flexion.  Hypesthesia of the lateral thigh would be more characteristic of a lesion of the lateral femoral cutaneous nerve.  Radiating medial thigh pain would suggest hip joint pathology or upper lumbar nerve root irritation.  Weakness in internal rotation is not a characteristic feature, and hypesthesia of the perineum would suggest possible involvement of the pudendal nerve.

 

REFERENCES: Byrd JWT: Thigh, hip, and pelvis, in Miller MD, Cooper DE, Warner JJP (eds): Review of Sports Medicine and Arthroscopy, ed 2.  Philadelphia, PA, WB Saunders, 2002,
pp 114-139.

Levin SM: Piriformis syndrome.  Orthopedics 2000;23:183-184.

 

 

 

 

74.     A 19-year-old football player who sustained three traumatic anterior shoulder dislocations underwent surgery to repair a Bankart lesion.  Nine months after surgery, examination reveals stability, elevation to 150 degrees, external rotation to 0 degrees with the elbow at his side and to 50 degrees at 90 degrees of abduction, and internal rotation to T12.  If his range of motion does not improve, he is at most risk for

 

1-         glenohumeral osteoarthritis.

2-         recurrent posterior subluxation.

3-         internal impingement syndrome.

4-         thoracic outlet syndrome.

5-         subscapularis tendon detachment.

 

PREFERRED RESPONSE: 1

 

DISCUSSION: Loss of external rotation can lead to degenerative joint disease following an anterior stabilization procedure.  A tight anterior capsule will prevent internal impingement.  Risk of thoracic outlet syndrome should not be increased.  Subscapularis detachment is a risk following open anterior repair; however, a gain in external rotation would be noted.  In time, this patient’s shoulder may show increased posterior glenohumeral wear but should not have symptoms of recurrent subluxation unless multidirectional instability is present.

 

REFERENCES: Hawkins RJ, Angelo RL: Glenohumeral osteoarthrosis: A late complication of the Putti-Platt repair.  J Bone Joint Surg Am 1990;72:1193-1197.

Norns TR: Complications following anterior instability repairs, in Bigliani LU (ed): Complication of Shoulder Surgery.  Baltimore, MD, Williams and Wilkins, 1993, pp 98-116.

 

 

 

 

75.     A 30-year-old man underwent an open Bankart repair with capsulorrhaphy for recurrent anterior instability 6 months ago.  In a recent fall, he described a hyperabduction and external rotation mechanism of injury.  He denies dislocating his shoulder.  He now has anterior shoulder pain, weakness, and the sensation of instability.  Examination reveals tenderness just lateral to the coracoid and bicipital groove.  An MRI scan is shown in Figure 31.  Management should now consist of

 

1-         immobilization for 3 weeks, followed by rehabilitation.

2-         open biceps tenodesis.

3-         arthroscopy with revision stabilization.

4-         arthroscopy with repair of the superior labrum.

5-         subscapularis repair.

 

PREFERRED RESPONSE: 5

 

DISCUSSION: Subscapularis tendon tears are being recognized with increasing frequency, and the mechanism of injury involves hyperabduction and external rotation.  The patient will have anterior shoulder pain and may report a sensation of instability.  Examination will reveal anterior shoulder tenderness over the lesser tuberosity and bicipital groove, and the Gerber lift-off test usually is positive.  The MRI scan shown here reveals an intact anterior labrum.  The subscapularis tendon is avulsed and retracted, with no evidence of the biceps tendon within the groove; this implies dislocation of the biceps, a common accompanying feature of a subscapularis tear.  This injury is also recognized as a complication after open anterior shoulder stabilizations where the subscapularis has been incised as part of the approach.  Therefore, the appropriate management involves repair of the subscapularis.  The injury does not represent a recurrence so immobilization or revision stabilization, which may be reasonable treatment for recurrent instability, is not indicated.  The findings are not consistent with a superior labral tear.

 

REFERENCES: Deutsch A, Altchek DW, Veltri DM, et al: Traumatic tears of the subscapularis tendon: Clinical diagnosis, magnetic resonance imaging findings, and operative treatment.  Am J Sports Med 1997;25:13-22.

Gerber C, et al: Isolated rupture of the subscapularis tendon.  J Bone Joint Surg Am 1996;78:1015-1023.

 

 

 

 

76.     What nerve is most at risk during placement of the anterolateral portal in elbow arthroscopy?

 

1-         Median

2-         Ulnar

3-         Radial

4-         Posterior interosseous

5-         Anterior interosseous

 

PREFERRED RESPONSE: 3

 

DISCUSSION: The radial nerve is only 4 mm from the anterolateral portal, while the median nerve is 11 mm away from the anteromedial portal.  The ulnar nerve is only at risk on the medial side of the elbow.  Anterior and posterior interosseous nerves are more distal within the forearm and are not in danger during portal placement.

 

REFERENCES: O’Driscoll SW, Morrey BF: Elbow arthroscopy, in Morrey BF (ed): The Elbow. Philadelphia, PA, Lippincott, Williams and Wilkins, 1994, pp 21-34.

Field LD, Altchek DW, Warren RF, O’Brien SJ, Skyhar MJ, Wickiewicz TL: Arthroscopic anatomy of the lateral elbow: A comparison of three portals.  Arthroscopy 1994;10:602-607.

 

 

 

 

77.     A 39-year-old man has anterior shoulder pain after landing on his abducted left shoulder while playing softball.  Examination reveals a stable glenohumeral joint, pain on passive external rotation of greater than 25 degrees, and pain and weakness on belly press (Napoleon’s) test.  An MRI scan is shown in Figure 32.  To provide maximum pain relief and return of function, management should include

 

1-         physical therapy to restore range of motion and rotator cuff strength.

2-         repair of the supraspinatus and biceps tenotomy.

3-         repair of the supraspinatus and biceps tenodesis.

4-         repair of the subscapularis and biceps tenotomy.

5-         repair of the subscapularis and biceps tenodesis.

 

PREFERRED RESPONSE: 5

 

DISCUSSION: The examination and MRI scan confirm a subscapularis rupture and dislocation of the long head of the biceps tendon.  The greatest return of function will result from repair of the subscapularis and tenodesis of the biceps tendon.  Physical therapy alone will result in inadequate healing of the subscapularis and will not address the biceps tendon.  While biceps tenotomy is an option, it will not provide the same level of pain relief and return of function as a tenodesis in a young, active man.  There is no evidence for a supraspinatus tear.

 

REFERENCES: Deutsch A, Altchek DW, Veltri DM, Potter HG, Warren RF: Traumatic tears of the subscapularis tendon: Clinical diagnosis, magnetic resonance imaging findings, and operative treatment.  Am J Sports Med 1997;25:13-22.

Gerber C, Hersche O, Farron A: Isolated rupture of the subscapularis tendon.  J Bone Joint Surg Am 1996;78:1015-1023.

 

 

 

 

78.     A 37-year-old racquet player had dominant shoulder pain for 1 year, and cortisone injections provided only temporary relief.  Because MRI findings did not reveal a rotator cuff tear, he underwent arthroscopic treatment including subacromial decompression and spur removal below the distal clavicle.  Three years following surgery, he now reports that the pain has returned.  What is the most likely cause of his pain?

 

1-         Acromioclavicular joint pathology

2-         Paralabral ganglion

3-         Villonodular synovitis

4-         Glenohumeral arthritis

5-         Superior labrum anterior and posterior lesion

 

PREFERRED RESPONSE: 1

 

DISCUSSION: Co-planing the distal clavicle may lead to painful acromioclavicular joints in up to 35% of patients; this is felt to be related to destabilizing the distal clavicle.  Intra-articular diagnosis of synovitis, degenerative joint disease, and superior labrum anterior and posterior lesions would have been identified at initial arthroscopy (not necessarily seen in open surgery).  Ganglions are seen on MRI.

 

REFERENCES: Fischer BW, Gross RM, McCarthy JA: Incidence of acromioclavicular joint complications after arthroscopic subacromial decompression.  Arthroscopy 1999;15:241-248.

Hazel RM, Tasto JP, Klassen J: Arthroscopic subacromial decompression: A 9-year follow-up.  Arthroscopy 1998;14:419.

Barber FA: Coplaning of the acromioclavicular joint.  Arthroscopy 2001;17:913-917.

 

 

 

 

79.     Figure 33 shows the radiograph of a 28-year-old avid golfer who has chronic right wrist pain.  Management should consist of

 

1-         cast immobilization.

2-         splinting with a bone stimulator.

3-         excision of the fracture fragment.

4-         arthroscopically assisted percutaneous fixation.

5-         trephination of the fibrous union.

 

PREFERRED RESPONSE: 3

 

DISCUSSION: The patient’s chronic symptoms are associated with a fracture of the base of the hook of the hamate; therefore, the treatment of choice is simple excision of the fracture fragment, with reasonable expectations of functional return.  Acute fractures may be difficult to treat because of the high incidence of nonunion, but once nonunion is discovered, nonsurgical management usually is unsuccessful.  Bone grafting may be a surgical alternative, but successful outcomes with percutaneous fixation or trephination of the fibrous union have not been reported.

 

REFERENCES: Geissler WB: Carpal fractures in athletes.  Clin Sports Med 2001;20:167-188.

Rettig ME, Dassa GL, Raskin KB, Melone CP Jr: Wrist fractures in the athlete: Distal radius and carpal fractures.  Clin Sports Med 1988;17:469-489.

 

 

 

 

80.    A 28-year-old hockey player has a shoulder deformity after being checked into the boards.  Examination reveals that swelling has improved, but there is tenderness along the distal clavicle.  Radiographs reveal a grade II acromioclavicular joint separation.  Initial management should consist of

 

1-         a sling, ice, and isometric exercises.

2-         a glenohumeral cortisone injection.

3-         surgical repair of the coracoclavicular ligaments.

4-         chin-ups and latissimus pull-down exercises.

5-         cross-chest stretches.

 

PREFERRED RESPONSE: 1

 

DISCUSSION: The most common shoulder injury in hockey players is to the acromioclavicular joint.  Early rest and control of pain and inflammation is the preferred management.  Surgery is reserved for patients with significant coracoclavicular disruption that has failed to respond to nonsurgical management.  Cross-chest stretches and overhead exercises may increase symptoms.  A cortisone injection within the glenohumeral joint will have little effect.

 

REFERENCES: Nuber GW, Bowen MK: Acromioclavicular joint injuries and distal clavicle fractures.  J Am Acad Orthop Surg 1997;5:11-18.

Weinstein DM, McCann PD, McIlveen SJ, Flatow EL, Bigliani LU: Surgical treatment of complete acromioclavicular dislocation.  Am J Sports Med 1995;23:324-331.

 

 

 

 

81.     Which of the following best describes athletic pubalgia?

 

1-         A syndrome of lower abdominal and adductor pain

2-         Painful symptoms emanating from the symphysis pubis

3-         Painful symptoms associated with dysfunction of the iliopsoas tendon

4-         Stress fracture of the pubic ramus

5-         Entrapment of the pudendal nerve

 

PREFERRED RESPONSE: 1

 

DISCUSSION: Athletic pubalgia refers to a distinct syndrome of lower abdominal and adductor pain that is mostly commonly seen in high performance male athletes.  This condition must be distinguished from others such as painful inflammation of the symphysis pubis, referred to as osteitis pubis.  Symptoms attributable to the iliopsoas tendon are most commonly associated with snapping of the tendon.  Stress fracture of the pubic ramus may cause symptoms in this area, but it is usually confirmed by imaging studies.  Neurapraxia of the pudendal nerve is associated with pressure from the seat in cycling sports and also as a complication associated with traction during surgical procedures.

 

REFERENCES: Meyers WC, Foley DP, Garrett WE, Lohnes JH, Mandlebaum BR: Management of severe lower abdominal or inguinal pain in high-performance athletes: PAIN (Performing Athletes with Abdominal or Inguinal Neuromuscular Pain Study Group).  Am J Sports Med 2000;28:2-8.

Albers SL, Spritzer CE, Garrett WE Jr, Meyers WC: MR findings in athletes with pubalgia.  Skeletal Radiol 2001;30:270-277.

 

 

 

 

82.     Figures 34a and 34b show the radiographs of a 28-year-old man who fell on his outstretched arm with significant force while mountain biking.  The nerve deficit most likely to occur would result in weakness of

 

1-         wrist extension.

2-         digital abduction.

3-         thumb flexion.

4-         thumb opposition.

5-         thumb extension.

 

PREFERRED RESPONSE: 5

 

DISCUSSION: The patient has a Monteggia fracture-dislocation (proximal ulnar fracture and radial head dislocation).  The posterior interosseous nerve branch of the radial nerve is the most likely to be injured and could result in weakness of thumb extension and finger metacarpal extension.  Considerably less likely are injuries to the more proximal radial nerve branches supplying the extensor carpi radialis longus and brevis, resulting in weak wrist extension; the ulnar nerve supplying the digital intrinsics, resulting in weak finger abduction; the anterior interosseous branch of the median nerve, resulting in weakness of the flexor pollicis longus; and the distal median nerve, resulting in weakness of thumb opposition.

 

REFERENCES: Bado JL: The Monteggia lesion.  Clin Orthop 1967;50:71-86.

Jupiter JB, Leibovic SJ, Ribbans W, Wilk RM: The posterior Monteggia lesion.  J Orthop Trauma 1991;5:395-402.

 

 

 

 

83.     Which of the following activities can improve posterior capsular contractures?

 

1-         Theraband exercises to strengthen the external rotator

2-         Latissimus pull-down exercises to the chest

3-         Seated rows

4-         Internal rotation stretch at 90 degrees abduction with scapular stabilization

5-         Bench press with wide grip

 

PREFERRED RESPONSE: 4

 

DISCUSSION: Posterior capsule stretching is performed in the cross-chest and behind the back positions.  Stretching in internal rotation in the abducted shoulder will further stretch the posterior capsule.  Wide grip stretch, and anterior capsule and strengthening exercises will not necessarily stretch the capsule.

 

REFERENCES: Ellenbacher TS: Shoulder internal and external rotation strength and range of motion of highly-skilled junior tennis players.  Isokinetic Exercise Sci 1992;2:1-8.

Kibler WB, McMullen J, Uhl J: Shoulder rehabilitation strategies, guidelines, and practice.  Op Tech Sports Med 2000;8:258-267.

 

 

 

 

84.     Figure 35 shows the radiograph of a 35-year-old weightlifter who has had pain with overhead lifts for the past 7 months.  Cortisone injections in the acromioclavicular joint provided only temporary relief.  A bone scan reveals increased activity of the acromioclavicular joint.  Treatment should now consist of

 

1-         rotator cuff interval closure.

2-         distal clavicle excision.

3-         superior labrum anterior and posterior repair.

4-         biceps tenodesis.

5-         thermal capsulorrhaphy.

 

PREFERRED RESPONSE: 2

 

DISCUSSION: Osteolysis of the distal clavicle is common in weightlifters; therefore, distal clavicle excision is the treatment of choice.  A subacromial decompression alone would not alleviate the acromioclavicular joint symptoms.  Interval closure, biceps degeneration, and superior labrum anterior and posterior repair would limit superior migration but would not explain the abnormal bone scan.  Thermal capsular shrinkage does not have a role here.

 

REFERENCES: Flatow EL, Cordasco FA, McCluskey GM, Bigliani LU: Arthroscopic resection of the distal clavicle via a superior portal: A critical quantitative radiographic assessment of bone removal.  Arthroscopy 1990;6:153-154.

Lyons FR, Rockwood CA: Osteolysis of the clavicle, in DeLee JC, Drez D (eds): Orthopaedic Sports Medicine.  Philadelphia, PA, WB Saunders, 1994, pp 541-546.

 

 

85.     Following reconstruction of the anterior cruciate ligament (ACL), which of the following rehabilitation exercises has the greatest potential to harm the graft?

 

1-         Active knee flexion from 45 to 90 degrees

2-         Active knee extension from 90 to 45 degrees

3-         Simultaneous isometric contraction of the quadriceps and hamstrings with a knee flexion angle between 30 and 60 degrees

4-         Isometric quadriceps contraction with a knee flexion angle between 0 and 30 degrees

5-         Isometric quadriceps contraction with a knee flexion angle between 60 and 90 degrees

 

PREFERRED RESPONSE: 4

 

DISCUSSION: Isometric quadriceps contraction between 15 and 30 degrees of flexion creates significant strain in the ACL and potential damage to the reconstructed graft.  Isolated quadriceps contraction with knee flexion of greater than 60 degrees, hamstring contraction at any angle of knee flexion, and active knee motion between 35 and 90 degrees of flexion create substantially less strain in the properly implanted ACL graft.

 

REFERENCES: Beynnon BD, Gleming BC, Johnson RL, Nichols CE, Renstrom PA, Pope MH: Anterior cruciate ligament strain behavior during rehabilitation exercises in vivo.  Am J Sports Med 1995;23:24-34.

Beynnon BD, Johnson RJ, Fleming BC, Stankewaich CJ, Renstrom PA, Nichols CE: The strain behavior of the anterior cruciate ligament during squatting and active flexion-extension: A comparison of an open and a closed kinetic chain exercise.  Am J Sports Med 1997;25:823-829.

 

 

 

 

 

86.     A young active patient with a complete isolated posterior cruciate ligament (PCL) tear undergoes a double bundle PCL reconstruction.  The tensioning pattern of the anterolateral (AL) and posteromedial (PM) bundles most likely to reproduce the most normal knee kinematics would be to tension

 

1-         both bundles at 45 degrees of flexion.   

2-         bundle AL at 45 degrees of flexion and bundle PM at 0 degrees of flexion.

3-         bundle AL at 45 degrees of flexion and bundle PM at 90 degrees of flexion.

4-         bundle AL at 90 degrees of flexion and bundle PM at 0 degrees of flexion.

5-         bundle AL at 90 degrees of flexion and bundle PM at 45 degrees of flexion.

 

PREFERRED RESPONSE: 4

 

DISCUSSION: During flexion and extension of the normal knee, the AL bundle of the PCL is taut in flexion, and the PM bundle is taut when the knee is near extension.  The AL bundle is approximately two times larger at its midsubstance, stiffer, and has a higher ultimate load than the PM bundle.  In vitro testing has demonstrated that by tensioning the AL bundle at 90 degrees of flexion and the PM bundle at 0 degrees of flexion, essentially normal knee kinematics are restored.  Tensioning the AL bundle at 45 degrees of flexion and the PM bundle at 0 degrees of flexion would result in increased laxity with flexion at 90+ degrees.  Tensioning the AL bundle at 90 degrees of flexion and the PM bundle at 45 degrees of flexion would result in increased laxity near extension.

 

REFERENCES: Harner CD, Janaushek MA, Kanamori A, Yagi M, Vogrin T, Woo SL: Biomechanical analysis of a double-bundle posterior cruciate ligament reconstruction.  Am J Sports Med 2000;28:144-151.

Mannor DA, Shearn JT, Grood ES, Noyes FR, Levy MS: Two-bundle posterior cruciate ligament reconstruction: An in vitro analysis of graft placement and tension.  Am J Sports Med 2000;28:833-845.

 

 

 

 

87.     Accurate evaluation of the upper portion of the subscapularis muscle is best accomplished with active internal rotation

 

1-         in adduction and the arm in external rotation.

2-         in adduction and the arm in neutral rotation.

3-         with the dorsum of the hand on the buttocks.

4-         with the dorsum of the hand on the midlumbar level.

5-         with the palm of the hand pressing against the belly.

 

PREFERRED RESPONSE: 5

 

DISCUSSION: Internal rotators of the shoulder include the subscapularis, pectoralis major, teres major, and latissimus dorsi muscles.  The subscapularis has two portions, with the upper portion receiving its innervation from the upper subscapular nerve (C5) and the lower portion from the lower subscapular nerve (C5-6).  The two tests commonly performed to isolate the internal rotation to the subscapularis muscle are the lift-off test and the belly press test.  Electromyographic findings have shown the lift-off test to be more accurate for the lower portion of the subscapularis and the belly press test to be more sensitive for the upper portion.

 

REFERENCES: Hintermeister RA, Lange GW, Schultheis JM, Bey MJ, Hawkins RJ: Electromyographic activity and applied load during shoulder rehabilitation exercises using elastic resistance.  Am J Sports Med 1998;26:210-220.

Greis PE, Kuhn JE, Schultheis J, Hintermeister R, Hawkins R: Validation of the lift-off test and analysis of subscapularis activity during maximal internal rotation.  Am J Sports Med 1996;24:589-593.

 

 

 

 

88.     During what phase of the throwing motion is the highest torque measured across the glenohumeral joint?

 

1-         Wind-up

2-         Cocking

3-         Early acceleration

4-         Late acceleration

5-         Follow through

 

PREFERRED RESPONSE: 5

 

DISCUSSION: Electromyography is used to evaluate muscular firing patterns about the shoulder during the throwing sequence.  The rotator cuff muscles and biceps are relatively inactive during the acceleration phase, whereas the pectoralis major, serratus anterior, latissimus dorsi, and subscapularis show highest activity.  By contrast, deceleration is accomplished by the rotator cuff musculature and the larger trunk muscles acting in concert to slow down the arm.  It is during this phase of follow through that the highest torque is measured secondary to eccentric muscle contraction.

 

REFERENCES: Jobe FW, Moynes DR, Tibone JE, Perry J: An EMG analysis of the shoulder in pitching: A second report.  Am J Sports Med 1984;12:218-220.

Pappas AM, Zawacki RM, Sulliva TJ: Biomechanics of baseball pitching: A preliminary report.  Am J Sports Med 1985;13:216-222.

Altcheck DW, Dines DM: Shoulder injuries in the throwing athlete.  J Am Acad Orthop Surg 1995;3:159-165.

 

 

 

 

89.     Figure 36 shows the radiograph of a 28-year-old man who injured his shoulder in a motocross race.  Management should consist of

 

1-         sling and swathe immobilization.

2-         early mobilization and strengthening exercises.

3-         an airplane abduction orthosis.

4-         stabilization with percutaneous smooth pins.

5-         open stabilization with coracoclavicular ligament repair and reconstruction.

 

PREFERRED RESPONSE: 5

 

DISCUSSION: Fractures of the distal one third of the clavicle have a high incidence of delayed union (45% to 67%) and nonunion (22% to 33%) with nonsurgical management.  Surgical stabilization with tension band techniques or a combination of plate and screw techniques is indicated, especially in young, active patients.  In this patient, significant displacement of the fracture implies injury to the coracoclavicular ligaments with a higher risk of delayed union or nonunion.  Various surgical treatments have been recommended, but the use of smooth wires is not indicated because of the potential for hardware migration.

 

REFERENCES: Jupiter JB, Ring D: Fractures of the clavicle, in Ianotti JP, Williams GR (eds): Disorders of the Shoulder: Diagnosis and Management, ed 1.  Philadelphia, PA, Lippincott, Williams and Wilkins, 1999, pp 709-736.

Kona J, Bosse MJ, Staehli JW, Rosseau RL: Type II distal clavicle fractures: A retrospective review of surgical treatment.  J Orthop Trauma 1990;4:115-120.

 

 

 

90.    Figure 37 shows the radiograph of a 21-year-old collegiate basketball player who has had mild midfoot aching for the past 4 months.  What is the best course of action?

 

1-         Functional bracing

2-         Cast immobilization with weight bearing permitted

3-         Cast immobilization with no weight bearing

4-         Open reduction and internal fixation

5-         Midfoot arthrodesis

 

PREFERRED RESPONSE: 4

 

DISCUSSION: A stress fracture of the navicular is considered a high-risk injury because of the incidence of nonunion.  If identified early, cast immobilization with no weight bearing is appropriate.  However, this patient is a high-caliber athlete who has had symptoms for 4 months.  Therefore, surgery is recommended to expedite recovery and optimize the chance of healing.

 

REFERENCES: Meyer SA, Saltaman CL, Albright JP: Stress fractures of the foot and ankle.  Clin Sports Med 1993;12:395-413.

Kahn JM, Fuller PJ, Burkner PD, et al: Outcome of conservative and surgical management of navicular stress fractures in athletes: Eighty-six cases proven with computerized tomography.  Am J Sports Med 1992;20:657-666.

Fitch KD, Blackwell JP, Gillmour WN: Application for non-union stress fractures of the tarsonavicular.  J Bone Joint Surg Br 1989;71:105.

 

 

 

91.     A 17-year-old high school gymnast who has peripatellar knee pain has been unable to practice on a consistent basis for the past 3 years.  She denies any specific injury events.  Physical therapy for modalities, quadriceps strengthening, and hamstring stretching provide temporary relief.  A trial of patellar taping significantly reduces her pain.  Examination reveals an 15-degree Q angle, moderate lateral facet tenderness, negative patellar apprehension, and the inability to evert the patella.  Radiographs show a moderate lateral patellar tilt.  Treatment should now consist of

 

1-         a lateral patellar restraining brace for practice and competition.    

2-         arthroscopic lateral retinacular release.

3-         open medial retinacular plication.

4-         medial tibial tubercle transfer.

5-         Maquet tibial tubercle elevation.

 

PREFERRED RESPONSE: 2

 

DISCUSSION: The patient has patellofemoral stress and a tight lateral retinaculum that has failed to respond to nonsurgical management; therefore, the most appropriate treatment includes an arthroscopic lateral retinacular release.  A patellar restraining brace may aggravate the peripatellar pain by increasing pressure on the lateral facet.  There is no evidence of patellar instability or significant malalignment; therefore, medial retinacular repair or a tibial tubercle transfer is not indicated.  A modified Maquet tibial tubercle elevation would be considered only for significant patellofemoral arthrosis.

 

REFERENCES: Gambardella RA: Techical pitfalls of patellofemoral surgery.  Clin Sports Med 1999;18:897-903.

Post WR: Clinical evaluation of patients with patellofemoral disorders.  Arthroscopy 1999;15:841-851.

 

 

 

 

92.    An active 55-year-old man who felt a sudden pop in the left heel while playing tennis 6 months ago was diagnosed with an ankle sprain around the time of injury.  He now reports calf atrophy and severe weakness with running.  Examination reveals a palpable defect in the Achilles tendon and only trace passive ankle flexion when the calf is squeezed.  At the time of surgery, an Achilles tendon defect of 6 cm cannot be approximated.  Surgical management of the Achilles tendon should include

 

1-         a local fascia turndown flap, followed by immobilization in a plantar flexed short leg cast for 12 weeks.

2-         multiple nonabsorbable sutures to bridge the gap and immobilization in a plantar flexed short leg cast for 12 weeks.

3-         a pull out wire and strict immobilization for 12 weeks.

4-         closure of the paratenon with a tight running suture, followed by rehabilitation.

5-         release of the flexor hallucis longus tendon at the Knot of Henry, followed by transfer through the calcaneus either alone or in combination with a V-Y advancement of the gastrocnemius.

 

PREFERRED RESPONSE: 5

 

DISCUSSION: Chronic or neglected Achilles tendon ruptures can present a surgical problem.  Ideally, end-to-end apposition of tendon should be attempted, but this should be accomplished without placing the foot in marked equinus.  A defect of greater than 5 cm requires the use of a tendon transfer either alone or in combination with a V-Y advancement of the gastrocnemius.  Because of its proximity to the Achilles tendon and its strength as a plantar flexor, the flexor hallucis longus is an ideal choice for this task.  Studies have shown that early active range-of-motion exercises after an Achilles tendon repair is beneficial for tendon healing and improved clinical outcomes.

 

REFERENCES: Myerson M: Achilles tendon ruptures.  Instr Course Lect 1999;48:219-230.

Mandelbaum BR, Myerson MS, Forster R: Achilles tendon ruptures: A new method of repair, early range of motion, and functional rehabilitation.  Am J Sports Med 1995;23:392-395.

 

 

93.    Figures 38a and 38b show the AP and lateral radiographs of a 12-year-old baseball pitcher who has pain in his right dominant elbow.  Management should consist of

 

1-         gentle range of motion and ultrasound.

2-         cast immobilization and a bone stimulator.

3-         elimination of offending activities and cross-training.

4-         arthroscopy with excision of the pathologic portion.

5-         MRI for assessment of accompanying ligamentous instability.

 

PREFERRED RESPONSE: 3

 

DISCUSSION: The patient has a stress fracture of the olecranon; therefore, the most appropriate management is modification of activities below the threshold of symptoms to allow for healing.  Ultrasound provides no benefit, and immobilization is not necessary.  MRI is not necessary because there is no associated ligamentous injury.  Arthroscopy is not indicated.

 

REFERENCES: Cabanela ME, Morrey BF: Fractures of the olecranon, in Morrey BF (ed): The Elbow and Its Disorders.  Philadelphia, PA, WB Saunders, 2000, pp 365-379.

Nuber GW, Diment MT: Olecranon stress fractures in throwers: A report of two cases and a review of the literature.  Clin Orthop 1992;278:58-61.

 

 

 

 

94.    What is the most reproducible landmark for the accurate anatomic placement of the tibial tunnel for an anterior cruciate ligament (ACL) reconstruction?

 

1-         Anterior border of the tibia

2-         Anterior border of the posterior cruciate ligament (PCL)

3-         Posterior border of the tibia

4-         Posterior border of the anterior horn of the lateral meniscus

5-         Posterior border of the anterior horn of the medial meniscus

 

PREFERRED RESPONSE: 2

 

DISCUSSION: The anterior border of the PCL is the most accurate and reproducible landmark for appropriate placement of the tibial tunnel for an ACL reconstruction.  The central sagittal insertion point of the ACL is consistently 10 to 11 mm anterior to the anterior border of the PCL ligament.  The anterior border of the tibia is not well visualized and does not serve as a reference point.  While the posterior border of the anterior horn of the lateral meniscus could be used as a reference point, it has twice the variability of the PCL reference point.  The posterior border of the tibia is difficult to identify and has greater variability than the PCL relative to the AP dimension of the proximal tibial surface.  The anterior horn of the medial meniscus is also more variable than the PCL.

 

REFERENCES: Hutchinson MR, Bae TS: Reproducibility of anatomic tibial landmarks for anterior cruciate ligament reconstructions.  Am J Sports Med 2001;29:777-780.

McGuire DA, Hendricks SD, Sanders HM: The relationship between anterior cruciate ligament reconstruction tibial tunnel location and the anterior aspect of the posterior cruciate ligament insertion.  Arthroscopy 1997;13:465-473.

 

 

 

95.     A 20-year-old football player sustains a dorsiflexion external rotation injury to his right ankle.  During sideline evaluation, which of the following findings best indicates a syndesmosis ankle sprain without diastasis?

 

1-         Tenderness over the anterior talofibular and calcaneofibular ligaments

2-         A positive dorsiflexion external rotation test

3-         Loss of passive range of motion

4-         Positive squeeze test

5-         Inability to single leg hop

 

PREFERRED RESPONSE: 5

 

DISCUSSION: The inability to single leg hop is considered the best indicator of a syndesmosis ankle sprain without diastasis.  Tenderness along the syndesmosis, the deltoid, or over the anterior talofibular ligament or anterior distal tibia/fibula may present later, following the initial injury.  The squeeze test and tenderness with dorsiflexion and external rotation may be positive but often are not present initially.  The best determinant for prediction of return to play is the amount of tenderness along the syndesmosis, measured from the distal fibula up the syndesmosis.

 

REFERENCES: Nussbaum ED, Hosea TM, et al: Prospective evaluation of syndesmosis ankle sprains without diastasis.  Am J Sports Med 2001;29:31-35.

Miller CD, Shelton WR, Barrett GR, et al: Deltoid and syndesmosis ligament injury of the ankle without fracture. Am J Sports Med 1985;23:746-750.

Amendola A: Controversies in diagnosis and management of syndesmosis injuries of the ankle.  Foot Ankle 1992;13:44-50.

 

 

 

96.    A 20-year-old college baseball pitcher reports the insidious onset of medial elbow pain.  Examination reveals medial elbow tenderness, a normal neurologic examination, and no obvious valgus laxity.  Plain radiographs are normal.  MRI scans are shown in Figures 39a and 39b.  Management should consist of

 

1-         repair of the medial collateral ligament.

2-         excision of the posterior olecranon osteophyte.

3-         debridement of the degenerative portion of the common flexor origin.

4-         autograft reconstruction of the medial collateral ligament.

5-         ulnar nerve transposition.

 

PREFERRED RESPONSE: 4

 

DISCUSSION: Throwers and in particular, pitchers, are prone to high valgus loads to the elbow.  A constellation of medial elbow pathology can develop, including medial epicondylitis, ulnar nerve neuritis, medial ulnar collateral ligament injuries, and posteromedial osteophytes of the olecranon.  The MRI scans show significant increases in signal intensing as well as fiber disruption of the medial collateral ligament, indicating a complete tear.  The common flexor origin shows a homogeneous signal and normal morphology.  Therefore, excision of posterior osteophytes and debridement of the common flexor origin are not indicated.  Likewise, this patient’s symptoms do not indicate ulnar nerve pathology; therefore ulnar nerve transposition is not indicated.  Primary repair of medial collateral ligament tears of the elbow lead to unpredictable results with an unacceptable rate of reoperation.  The most predictable result in treating this high-demand athlete is reconstruction of the medial collateral ligament with autogenous tissue.

 

REFERENCES: Norris TR (ed): Athletic Injuries of the Elbow.  Rosemont, IL, American Academy of Orthopaedic Surgeons, 1997, pp 311-323.

Hyman J, Breazeale NM, Altchek DW: Valgus instability of the elbow in athletes.  Clin Sports Med 2001;20:11-24.

 

 

 

 

97.     What is the most common arthroscopic finding of internal impingement in an
overhead athlete?

 

1-         Loose body

2-         Type III acromion

3-         Bankart lesion

4-         Rotator cuff articular side tear

5-         Biceps tendon fraying

 

PREFERRED RESPONSE: 4

 

DISCUSSION: Internal impingement occurs when the articular side of the supraspinatus abrades against the posterior superior glenoid in the cocking position.  Damage may include a posterior labral tear where the contact occurs, not anteriorly as in a Bankart lesion.  Biceps fraying and acromion spurs are more commonly seen in extrinsic impingement.  Loose bodies may occur from multiple lesions associated with instability and articular cartilage disorders but are uncommon in internal impingement.

 

REFERENCES: Jobe CM: Posterior superior impingement of the rotator cuff on the glenoid rim as a cause of shoulder pain in the overhead athlete.  Arthroscopy 1993;9:697-699.

McFarland EG, Hsu C, Neir C, O’Neil O: Internal impingement of the shoulder: A clinical and arthroscopic analysis.  J Shoulder Elbow Surg 1999;8:458-460.

 

 

 

 

98.     A 16-year-old high school football player who sustained an acute forceful dorsiflexion ankle injury reported that he felt a pop and then noted immediate swelling over the lateral malleolus.  Examination 24 hours later reveals moderate swelling and tenderness along the lateral malleolus.  The external rotation, squeeze, anterior drawer, and talar tilt tests are negative.  Subluxation of the peroneal tendons is palpable over the peroneal groove of the fibula.  Radiographs reveal a small cortical avulsion off the distal rim of the fibula.  The stress views show no instability.  Initial management for this injury should include

 

1-         a lace-up ankle splint and progressive activities.

2-         anatomic repair of the anterior talofibular and calcaneofibular ligaments.

3-         Kirschner wire and tension band fixation of the cortical avulsion fracture off the fibula.

4-         local ligament transfer and reconstruction of the lateral ankle ligaments.

5-         protected weight bearing and a short leg cast for 6 weeks.

 

PREFERRED RESPONSE: 5

 

DISCUSSION: The patient has an acute peroneal tendon dislocation.  The evaluation for syndesmotic injury and lateral ankle instability is negative.  The cortical avulsion off the distal tip of the lateral malleolus, a rim fracture, is characteristic of peroneal tendon dislocations.  The sensation of apprehension or frank subluxation of the peroneal tendons with active dorsiflexion of the foot while the foot is held in plantar flexion confirms the diagnosis.  Based on these findings, initial management should consist of cast immobilization and protected weight bearing.  If a recurrent or chronic condition develops, surgery is the most reliable treatment option.  

 

REFERENCES: Arrowsmith SR, Fleming LL, Allman FL: Traumatic dislocations of the peroneal tendons.  Am J Sports Med 1983;11:142-146.

Marti R: Dislocation of the peroneal tendons.  Am J Sports Med 1977;5:19-22.

 

 

 

 

99.    The essential lesion responsible for posterolateral rotatory instability of the elbow is disruption of the

 

1-         lateral ulnar collateral ligament.

2-         medial collateral ligament.

3-         radial collateral ligament.

4-         annular ligament.

5-         posterolateral capsule.

 

PREFERRED RESPONSE: 1

 

DISCUSSION: Posterolateral rotatory instability (PLRI) of the elbow represents a three-dimensional injury pattern of rotational displacement of the ulna from the trochlea and the radius from the capitellum.  The ulna supinates (externally rotates) past its normal limit and the radiocapitellar joint subluxates posterolaterally, permitting the coronoid process to slide beneath the trochlea.  In cadaver studies, the lateral ulnar collateral ligament has been shown to be the essential lesion responsible for PLRI.  The medial collateral ligament (of which the anterior bundle is the most important) is the primary restraint to valgus instability.  The posterolateral capsule and radial collateral ligament may be disrupted in a complete posterolateral dislocation but are not essential injuries for PLRI.  The primary function of the annular ligament is to stabilize the proximal radioulnar joint.

 

REFERENCES: O’Driscoll SW, Jupiter JB, King GJW, Hotchkiss RN, Morrey BF: The unstable elbow.  J Bone Joint Surg Am 2000;82:724-738.

Olsen BS, Sojbjerg JO, Dalstra M, Sneppen O: Kinematics of the lateral constraints of the elbow.  J Shoulder Elbow Surg 1996;5:333-341.

O’Driscoll SW, Morrey BF, Korinek S, An KN: Elbow subluxations and dislocation: A spectrum of instability.  Clin Orthop 1992;280:186-197.

 

 

 

 

100.  When evaluating articular cartilage, what extracellular matrix component is most closely associated with the deep calcified cartilage zone?

 

1-         Collagen type I

2-         Collagen type II

3-         Collagen type X

4-         Proteoglycan aggrecan

5-         Hyaluronic acid

 

PREFERRED RESPONSE: 3

 

DISCUSSION: Collagen type X is produced only by hypertrophic chondrocytes during enchondral ossification (growth plate, fracture callus, heterotopic ossification) and is associated with calcification of cartilage in the deep zone of articular cartilage.  Collagen type I is the predominant collagen in bone, ligament, and tendon.  Collagen type II is the predominant collagen in articular cartilage.  Proteoglycan aggrecan and hyaluronic acid are components of the extracellular matrix and are involved in the compressive strength characteristics of articular cartilage.

 

REFERENCES: Buckwalter JA, Mankin HJ: Articular cartilage: Tissue design and chondrocyte matrix interactions.  Instr Course Lect 1998;47:477-486.

Poole AR, Kojima J, Yasuda T, Mwale F, Kobayasai M, Laverty S: Composition and structure of articular cartilage: A template for tissue repair.  Clin Orthop 2001;391:S26-S33.

 

 

 

 

FOR ALL MCQS CLICK THE LINK ORTHO MCQ BANK